Download as pdf or txt
Download as pdf or txt
You are on page 1of 52

Neurocritical 

Care Review 
Practice Questions 
Section 1. 

1. Question: An 18‐year‐old man is admitted to the neurologic ICU after a motor vehicle collision. On
examination, his eyes are closed and do not open with noxious stimulation. He is intubated and can pull
his left arm away from nail bed pressure. His right arm flexes when central pain is applied. His Glasgow
Coma Scale score is

A. 3
B. 5
C. 6
D. 7
E. 10

Answer: C. 6 

Rationale: The Glasgow Coma Scale (GCS) evaluates 3 areas of the neurologic examination—motor 
response, verbal response, and eye opening—and ranges from 3 to 15 total points. Six points are 
available for motor score: commands 6, localization 5, withdrawal 4, flexion 3, extension 2, no response 
1. Five points are available for verbal score: oriented 5, confused 4, inappropriate responses or words 3,
incomprehensible speech 2, no verbal response or intubated 1. Four points are available for eye score:
eyes open spontaneously 4, eyes open to voice 3, eyes open to pain 2, no eye opening 1. This patient’s
best motor response was to pull away from the examiner, which is withdrawal and scores 4 points. He is
intubated and therefore has no verbal response, which scores 1 point. He was not able to open his eyes
so he scores 1 point for eye opening. The total Glasgow Coma Scale score is 6.

Reference: 
1.) Teasdale G, Jennett B. Assessment of coma and impaired consciousness. A practical 
scale. Lancet. 1974 Jul 13;2(7872):81‐84. 

2. Question: A 59‐year‐old man with hypertension and obesity is admitted for spinal decompression for
lumbar stenosis. He was unresponsive on arrival. He can briefly open his eyes to painful stimuli. Pupils
and face are symmetric. He does not follow directions but is able to push the examiner away with both
upper extremities. When he is not being stimulated his eyes remain closed. This patient’s examination is
best described as

A. fluctuating examination consistent with delirium
B. focal neurologic examination due to loss of ability to follow directions
C. focal examination most likely related to analgesic medication administration
D. nonfocal examination most likely due to acute stroke
E. nonfocal examination most likely related to analgesic medications

Answer: E. nonfocal examination most likely related to analgesic medications 

Rationale: The neurologic examination can identify focal abnormalities in cranial nerve function, 
corticospinal tract, peripheral nerves, and brainstem. These domains are evaluated systematically in the 

1
Neurocritical Care Review 
Practice Questions 
Section 1. 

mental status, motor, and cranial nerve examinations. A focal deficit suggests a structural lesion or area 
of regional dysfunction, thus prompting workup of acute ischemic stroke, acute intracranial 
hemorrhage, or structural lesion. Nonfocal examinations are more likely to be related to a postictal 
state, metabolic derangements, toxidromes, or medication effects. This patient has symmetric cranial 
nerves as tested with symmetric strength of his extremities consistent with a nonfocal examination. He 
does not have findings consistent with delirium, including fluctuating and acute onset of disturbances 
memory, orientation, attention, or consciousness. Inability to follow commands is not consistent with a 
focal examination. 

Reference:  
1.) Caplan LR, Searls DE, Hon FKS. Cerebrovascular disease. Med Clin North Am. 2009 Mar;93(2):353‐369, 
viii. 

3. Question: A 75‐year‐old man presents to the emergency department with profound right‐sided
weakness and expressive aphasia. His wife witnessed the onset of his symptoms 4 hours ago.
Noncontrast head CT is normal. Vital signs are: temperature 37 °C (98.5 °F), blood pressure 190/95 mm
Hg, heart rate 98 beats/min, respiratory rate 16/min, and oxygen saturation 94% on 2 L nasal cannula.
Which of the following is the most appropriate next intervention?

A. Administer IV alteplase.
B. Administer aspirin.
C. Administer labetalol, 20 mg IV.
D. Consult the neuroendovascular team for emergent thrombectomy.
E. Obtain CT perfusion.

Answer: C. Administer labetalol, 20 mg IV. 

Rationale: This patient has acute ischemic stroke beyond the standard 3‐hour window of last known 
well. The ECASS III trial showed that IV alteplase up to 4.5 hours after symptom onset was beneficial.This 
patient meets this criterion; however, because the blood pressure is more than the allowable blood 
pressure for IV alteplase, labetalol should be administered first to achieve the goal of less than 185/105 
mm Hg. Patients excluded from alteplase in the 4.5‐hour window previously comprised patients taking 
warfarin, those with a combined history of diabetes mellitus and previous ischemic stroke and very 
severe strokes with an NIH Stroke Scale score greater than 25; however, the most recent American 
Heart Association guidelines included these population after data analysis since the exclusion of these 
patients may not have been justified. Because this patient is a candidate for IV thrombolysis, this should 
be prioritized before calling the endovascular team. Aspirin must be held for 24 hours after IV alteplase 
administration because of the risk of hemorrhage. CT perfusion may guide further endovascular 
management but should not be performed until after treatment with IV alteplase. 

References:  
1.) Warner JJ, Harrington RA, Sacco RL, Elkind MSV. Guidelines for the early management of patients 
with acute ischemic stroke: 2019 update to the 2018 guidelines for the early management of acute 
ischemic stroke. Stroke. 2019 Dec;50(12):3331‐3332. 

2
Neurocritical Care Review 
Practice Questions 
Section 1. 
 
 
2.) Hacke W, Kaste M, Bluhmki E, et al; ECASS Investigators. Thrombolysis with alteplase 3 to 4.5 hours 
after acute ischemic stroke. N Engl J Med. 2008 Sep 25;359(13):1317‐1329. 
 
4. Question: A 55‐year‐old woman presents to the emergency department with a headache and 
diplopia. On examination, she has right eye ptosis, right pupillary dilation, and limitation of eye 
movement in the right eye on superior and medial gaze. Her pupillary dilation is likely caused by 
 
A. compression of the levator palpebrae superioris 
B. dysfunction of the orbicularis oculi 
C. dysfunction of the short ciliary nerves 
D. increased sympathetic tone accompanying migraine headache 
E. interruption of fibers in the sympathetic cervical ganglion 
 
Answer: C. dysfunction of the short ciliary nerves 
 
Rationale: This patient presents with dysfunction of the third cranial (oculomotor) nerve, which has 
purely motor function and is responsible for the innervation of all extraocular muscles of the eye except 
the superior oblique and lateral rectus. Loss of function of the superior and medial rectus muscles in one 
eye causes deconjugate gaze and diplopia. The trigeminal ganglion is the sensory ganglion for the 
trigeminal nerve. Dysfunction leads to loss of sensation of the face. Dysfunction of the levator palpebrae 
superioris leads to ptosis. Loss of function in the constrictor pupillae and ciliary muscles leads to 
pupillary dilation. Overaction of the sympathetic cervical ganglion leads to increased sympathetic 
activity, including pupillary dilation. 
 
Reference: 
1.) Disorder of ocular movement and pupillary function. In: Ropper A, Samuels M, Klein J. Adams and 
Victor’s Principles of Neurology. 10th ed. McGraw‐Hill Education; 2014:chapt 14. 
 
 
5. Question: MRI is superior to CT in traumatic brain injury in its 
 
A. short acquisition time 
B. visualization of acute hemorrhage 
C. visualization of parenchymal injury 
D. visualization of skull fracture 
 
Answer: C. visualization of parenchymal injury 
 
Rationale: Head CT is usually the modality of choice in the acute setting because of its ready availability, 
inexpensive cost, and short acquisition times. CT is superior to MRI in visualizing hemorrhage and bone 
fractures but is inadequate for detecting some types of parenchymal injury such as diffuse axonal injury. 
 
References: 

 
3
Neurocritical Care Review 
Practice Questions 
Section 1. 
 
1.) Carney N, Totten AM, O’Reilly C, et al. Guidelines for the management of severe traumatic brain 
injury, fourth edition. Neurosurgery. 2017 Jan 1;80(1):6‐15. 
 
2.) Jolly AE, Bălăeţ M, Azor A, et al. Detecting axonal injury in individual patients after traumatic brain 
injury. Brain. 2021 Feb 12;144(1):92‐113. 
 
 
6. Question. A 32‐year‐old woman sustains multiple injuries including bifrontal cerebral contusions after 
being struck by a car as a pedestrian. Glasgow Coma Scale score was 6 on admission, and a fiberoptic 
intracranial pressure (ICP) monitor was placed. ICP has been sustained at an elevated value of 35 mm Hg 
for the past 20 minutes. The clinical team chooses to treat the intracranial hypertension with hypertonic 
saline. To avoid renal injury, the clinician should 
 
A. administer hypertonic saline through a central venous line rather than a peripheral IV line 
B. avoid adding potassium to IV fluids 
C. avoid exceeding serum sodium level of 160 mEq/L or serum osmolarity of 320 mmol/kg 
D. initiate a continuous infusion of 3% hypertonic saline 
 
Answer: C. avoid exceeding serum sodium level of 160 mEq/L or serum osmolarity of 320 mmol/kg 
 
Rationale: Serial serum sodium and serum osmolarity should be checked to ensure that sodium is 
maintained under goal. Classic studies suggest that prolonged exposure to serum sodium levels above 
160 mEq/L and/or serum osmolarity above 320 mmol/kg are associated with acute kidney injury. 
Osmotherapy is generally administered as bolus dosing to improve intracranial pressure effect, allowing 
clearance between doses to avoid renal injury. Potassium is often added to IV fluids because 
hypokalemia is a frequent side effect of induced hypernatremia but does not prevent renal injury. Route 
of administration of 3% hypertonic saline does not impact renal function. 
 
Reference:  
1.) Hinson HE, Stein D, Sheth KN. Hypertonic saline and mannitol therapy in critical care neurology. J 
Intensive Care Med. 2013 Jan‐Feb;28(1):3‐11. 
 
 
7. Question: A 72‐year‐old woman with a previous medical history of breast cancer in remission is 
admitted to the neuro‐ICU with a modified Fisher 2, Hunt and Hess 3, subarachnoid hemorrhage. Head 
CT angiogram reveals a 5‐mm anterior communicating artery aneurysm. Which of the following 
statements is true regarding this patient’s aneurysm treatment in regard to clipping or coil 
embolization? 
 
A. Ten‐year outcomes are better if the aneurysm is clipped. 
B. Aneurysm clipping results in lower epilepsy risk. 
C. Aneurysm recurrence rate is lower with coiling. 
D. Aneurysm securement with clipping or coiling should be performed at least 72 hours after 
admission. 
E. Chances of a survival and freedom from disability at 1 year are higher with coiling. 

 
4
Neurocritical Care Review 
Practice Questions 
Section 1. 
 
 
Answer: E. Chances of a survival and freedom from disability at 1 year are higher with coiling. 
 
Rationale: The International Subarachnoid Aneurysm Trial (ISAT) demonstrated that patients with 
subarachnoid hemorrhage (SAH) who underwent endovascular coiling for the aneurysm had a higher 
chance of disability‐free survival at 1 year compared with those who underwent surgical clipping. 
Clipping is associated with a higher epilepsy risk. Aneurysm recurrence rate is higher if treated with 
coiling compared with clipping. Despite this, the ISAT trial demonstrated that 10‐year outcomes are still 
better with coiling than with clipping. American Heart Association and Neurocritical Care Society 
guidelines recommend aneurysm treatment as early as possible to reduce the rate of rebleeding after 
SAH. 
 
References: 
1.) Connolly ES Jr, Rabinstein AA, Carhuapoma JR, et al; American Heart Association Stroke Council; 
Council on Cardiovascular Radiology and Intervention; Council on Cardiovascular Nursing; Council on 
Cardiovascular Surgery and Anesthesia; Council on Clinical Cardiology. Guidelines for the management 
of aneurysmal subarachnoid hemorrhage: a guideline for healthcare professionals from the American 
Heart Association/American Stroke Association. Stroke. 2012 Jun;43(6):1711‐1737. 
 
2.) Diringer MN, Bleck TP, Hemphill JC 3rd, et al; Neurocritical Care Society. Critical care management of 
patients following aneurysmal subarachnoid hemorrhage: recommendations from the Neurocritical Care 
Society's Multidisciplinary Consensus Conference. Neurocrit Care. 2011 Sep;15(2):211‐240. 
 
3.) Molyneux A, Kerr R; International Subarachnoid Aneurysm Trial (ISAT) Collaborative Group,et al. 
International Subarachnoid Aneurysm Trial (ISAT) of neurosurgical clipping versus endovascular coiling in 
2143 patients with ruptured intracranial aneurysms: a randomized trial. J Stroke Cerebrovasc Dis. 2002 
Nov‐Dec;11(6):304‐314. 
 
4.) Molyneux AJ, Birks J, Clarke A, Sneade M, Kerr RS. The durability of endovascular coiling versus 
neurosurgical clipping of ruptured cerebral aneurysms: 18‐year follow‐up of the UK cohort of the 
International Subarachnoid Aneurysm Trial (ISAT). Lancet. 2015 Feb 21;385(9969):691‐697. 
 
 
 
 
 
 
 
 
 
 
 
 
 

 
5
Neurocritical Care Review 
Practice Questions 
Section 1. 
 
8. Question: A 51‐year‐old man with a history of hypertension, smoking, hyperlipidemia, deep vein 
thrombosis/pulmonary embolism, and prior myocardial infarction was admitted with a modified Fisher 
4, Hunt and Hess 5, subarachnoid hemorrhage (SAH) due to a 7‐mm left middle cerebral artery 
aneurysm seen on head CT angiogram. He was intubated for airway protection and an external 
ventricular drain was placed by neurosurgery for acute hydrocephalus. He has hypoxia and is too 
unstable for transfer to the angiography suite for a cerebral angiogram. He is hypertensive, with blood 
pressure 185/92 mm Hg. Which of the following statements is true regarding management of his 
rebleeding risk? 
 
A. Administration of antifibrinolytics should be considered to reduce rebleeding risk. 
B. IV hydralazine is most appropriate to target a goal systolic blood pressure (SBP) of less than 160 
mm Hg. 
C. Mortality rate is approximately 10% if rebleeding occurs. 
D. Nicardipine infusion should be started to target a goal SBP of less than 160 mm Hg. 
E. Risk of rebleeding is highest 7 days after SAH onset. 
 
Answer: D. Nicardipine infusion should be started to target a goal SBP of less than 160 mm Hg. 
 
Rationale: A continuous infusion of an IV antihypertensive medication is preferred over bolus 
medications to prevent wide fluctuations in blood pressure that may precipitate aneurysm rerupture. 
Current guidelines recommend targeting a systolic blood pressure goal of less than 160 mm Hg. 
Additionally, IV hydralazine can cause rebound hypertension, potentially contributing to wide 
fluctuations in blood pressure. The mortality rate associated with aneurysm rebleeding is 20%‐60%. The 
risk of rebleeding is highest within the first 72 hours after subarachnoid hemorrhage, with the majority 
occurring in the first 6 hours. A short‐term (< 72 hours) course of antifibrinolytics can be considered in 
this patient to reduce the risk of aneurysm rerupture since definitive aneurysm treatment is delayed; 
however, because of his history of deep vein thrombosis/pulmonary embolism and myocardial 
infarction, he is at very high risk for a thrombotic complication from antifibrinolytics. 
 
References:  
1.) Macdonald RL, Schweizer TA. Spontaneous subarachnoid haemorrhage. Lancet. 2017 Feb 
11;389(10069):655‐666. 
 
2.) Tang C, Zhang TS, Zhou LF. Risk factors for rebleeding of aneurysmal subarachnoid hemorrhage: a 
meta‐analysis. PLoS One. 2014 Jun 9;9(6):e99536. 
 
3.) Starke RM, Kim GH, Fernandez A, et al. Impact of a protocol for acute antifibrinolytic therapy on 
aneurysm rebleeding after subarachnoid hemorrhage. Stroke. 2008 Sep;39(9):2617‐2621. 2008 
Sep;39(9):2617‐2621. 
 
4.) Muehlschlegel S. Subarachnoid hemorrhage. Continuum (Minneap Minn). 2018 Dec;24(6):1623‐1657. 
 
 
9. Question: A 20‐year‐old man was found unconscious on the sidewalk. Emergency medical services 
placed him in a cervical collar and brought him to the emergency department. He is unresponsive, so a 

 
6
Neurocritical Care Review 
Practice Questions 
Section 1. 
 
neurologic examination is requested. Which of the following techniques is most appropriate for testing 
the vestibular system? 
 
A. Fill a syringe with ice water and slowly irrigate into the right external auditory canal, watching 
for eye movement to the left. 
B. Fill a syringe with ice water and slowly irrigate into the right external auditory canal, watching 
for eye movement to the right. 
C. Perform the oculocephalic (doll’s eye) reflex. 
D. Place the head at 30 degrees and perform the Dix‐Hallpike maneuver. 
E. Shine a flashlight in the right eye and observe the left pupil for constriction. 
 
Answer: B. Fill a syringe with ice water and slowly irrigate into the right external auditory canal, 
watching for eye movement to the right. 
 
Rationale: The vestibular system can be evaluated with cold water caloric testing, performed by slowly 
irrigating cold water into the external auditory canal and watching for the slow component of nystagmus 
to move the eyes toward the cold stimulus, which occurs from cooling of the endolymph in the 
semicircular canal. The cooled endolymph is denser and moves inferiorly, thus moving the hair cells of 
the cochlea and activating efferent pathways of the vestibulocochlear nerve to the brainstem. The Dix‐
Hallpike maneuver is performed in awake patients being evaluated for vestibular neuronitis. The 
oculocephalic (doll’s eye) reflex tests cranial nerves III and VI and should not be performed on patients 
in cervical collars or with known cervical injury. Pupillary constriction tests cranial nerve II and III 
responses. 
 
References: 
1.) Campbell WW. DeJong’s The Neurologic Examination. 7th ed. Lippincott Williams & Wilkins; 2012. 
 
10. Question: A 55‐year‐old woman with a history of hypertension, coronary artery disease, and 
previous strokes (treated with aspirin and clopidogrel) was found unconscious on the floor. In the 
emergency department (ED), blood pressure is 188/97 mm Hg, and Glasgow Coma Scale score is 7, 
necessitating intubation for airway protection. On examination, she has right hemiplegia, left pupil is 7 
mm and unreactive, right pupil is 4 mm and reactive, and she flexes on the left and extends on the right 
to noxious stimuli. Noncontrast head CT reveals an acute, left frontotemporal hemorrhage with 
effacement of the basal cisterns and uncal herniation. She is prepared for emergent craniectomy and 
hematoma evacuation. Platelet mapping using thromboelastography shows arachidonic acid and 
adenosine diphosphate pathway inhibition rates of 95%. The ED physician asks whether the patient 
should receive platelet transfusion and/or desmopressin for antiplatelet effect reversal. Which of the 
following is the most appropriate response? 
 
A. A combination of platelet transfusion and desmopressin is indicated. 
B. Neither platelet transfusion nor desmopressin is indicated. 
C. Only desmopressin is indicated. 
D. Only platelet transfusion is indicated. 
 
Answer: A. A combination of platelet transfusion and desmopressin is indicated. 

 
7
Neurocritical Care Review 
Practice Questions 
Section 1. 
 
Rationale: Thromboelastography (TEG)‐guided intraoperative and postoperative platelet transfusion for 
inhibition of rates greater than 89% on both the ADP‐ and arachidonic acid‐dependent pathways has 
been shown to confer benefit in intraoperative blood loss, total blood transfusion volume, residual 
postoperative hematoma volume, surgical drain output volume, recurrence rate of secondary bleeding, 
and need for repeat surgery. Multiple retrospective studies have shown that desmopressin improved 
several tests of platelet function, including bleeding time, ADP‐activated platelet function analyzer, ADP‐
dependent platelet activity on TEG, collagen/ADP closure time, and collagen/epinephrine closure time, 
for those on aspirin/COX‐1 inhibitors or ADP receptor inhibitors, compared with those who received no 
desmopressin. Also, desmopressin has been shown to significantly reduce blood loss and improve 
thrombus formation in those undergoing cardiac surgery who were on acetylsalicylic acid 
preoperatively. This patient is undergoing emergent neurosurgical intervention with significant 
pharmacologic inhibition of platelet function. Using a combination of desmopressin and platelet 
transfusions for the operative course is most appropriate. If time is a barrier to performing platelet 
function testing, empiric platelet transfusion for surgery is still reasonable. 
 
References:  
1.) Zhou H, Chen L, He H. Intraoperative and postoperative effects of TEG‐guided platelet transfusion on 
antiplatelet drug‐related intracerebral hemorrhage patients. Exp Ther Med. 2019 Mar;17(3):2263‐2267. 
 
2.) Desborough MJR, Oakland KA, Landoni G, et al. Desmopressin for treatment of platelet dysfunction 
and reversal of antiplatelet agents: a systematic review and meta‐analysis of randomized controlled 
trials. J Thromb Haemost. 2017 Feb;15(2):263‐272. 
 
3.) Frontera J, Lewin JJ 3rd, Rabinstein AA, et al. Guideline for reversal of antithrombotics in intracranial 
hemorrhage: a statement for healthcare professionals from the Neurocritical Care Society and Society of 
Critical Care Medicine. Neurocrit Care. 2016 Feb;24(1):6‐46. 
 
 
11. Question: A 27‐year‐old man is being treated in the ICU for diffuse axonal injury sustained in a high‐
speed motor vehicle collision. He is in a coma and is being monitored with an intracranial pressure (ICP) 
monitor. The waveform generated by the monitor shows an elevated second peak on the ICP tracing. 
The bedside nurse alerts the clinical team that the patient’s right pupil is now dilated and sluggishly 
reactive while the left pupil is small and briskly reactive. As the team orders a head CT to confirm the 
diagnosis, which of the following medications is most appropriate to address the new clinical 
deterioration? 
 
A. Dexamethasone 
B. Furosemide 
C. Mannitol 
D. Urea 
 
Answer: C. Mannitol 
 
Rationale: This patient has likely developed a herniation syndrome related to worsening cerebral 
edema, as indicated by intracranial waveform tracing. Mannitol or hypertonic saline are both acceptable 

 
8
Neurocritical Care Review 
Practice Questions 
Section 1. 
 
options to lower intracranial pressure (ICP) and/or reverse a clinical herniation syndrome. Urea 
produces an osmotic effect but much less than mannitol because of its lower coefficient of reflection. 
Furosemide would have a diuretic effect and possibly raise serum sodium, but this would not be 
expected to lower ICP. Steroids are not indicated for cerebral edema caused by traumatic brain injury. 
 
References:  
1.) Hinson HE, Stein D, Sheth KN. Hypertonic saline and mannitol therapy in critical care neurology. J 
Intensive Care Med. 2013 Jan‐Feb;28(1):3‐11.  
 
2.) Carney N, Totten AM, O’Reilly C, et al. Guidelines for the management of severe traumatic brain 
injury, fourth edition. Neurosurgery. 2017 Jan 1;80(1):6‐15. 
 
 
12. Question: A 60‐year‐old man is in the cardiovascular ICU recovering from a recent coronary artery 
bypass graft. On postoperative day 2 he is left in a chair during the physical therapist’s lunch break. 
When the nurse checks on the patient an hour later, he is in the chair awake and following commands 
but dysarthric with left‐sided weakness, left facial droop, left hemineglect, and right gaze preference. 
Vital signs are: temperature 36.7 °C (98 °F), blood pressure 180/85 mm Hg, heart rate 75 beats/min, 
respiratory rate 16 breaths/min, and oxygen saturation 93% on 2 L nasal cannula. Head CT is negative, 
and head CT angiogram shows a right middle cerebral artery occlusion. Which of the following is the 
most appropriate next intervention? 
 
A. IV thrombolysis administration 
B. Neuroendovascular team consultation for emergent evaluation for mechanical thrombectomy 
C. Neurosurgery consultation for potential decompressive hemicraniectomy 
D. Continuous EEG monitoring 
E. Emergent CT perfusion 
 
Answer: B. Neuroendovascular team consultation for emergent evaluation for mechanical 
thrombectomy 
 
Rationale: This patient with a post‐coronary artery bypass graft likely had a cardioembolic stroke 
resulting in the occlusion of the right middle cerebral artery. He is not a candidate for IV thrombolysis 
because of the recent procedure but he is a candidate for mechanical thrombectomy. Emergent CT 
perfusion is not needed because he is within the window for thrombectomy. Continuous EEG is not 
needed at this time because seizures are lower on the differential diagnosis in this clinical setting of an 
awake patient with focal deficits. Because he is still a candidate for reperfusion therapy, there is 
currently no role for decompressive hemicraniectomy in this acute phase. 
 
Reference:  
1.) Warner JJ, Harrington RA, Sacco RL, Elkind MSV. Guidelines for the early management of patients 
with acute ischemic stroke: 2019 update to the 2018 guidelines for the early management of acute 
ischemic stroke. Stroke. 2019 Dec;50(12):3331‐3332. 
 
 

 
9
Neurocritical Care Review 
Practice Questions 
Section 1. 
 
13. Question: The 4 main blood vessels that course through the neck to supply blood to the brain are 
 
A. Right anterior cerebral artery, left anterior cerebral artery, right vertebral artery, left vertebral 
artery 
B. Right cerebral artery, left cerebral artery, right vertebral artery, left vertebral artery 
C. Right external carotid artery, left external carotid artery, right vertebral artery, left vertebral 
artery 
D. Right internal carotid artery, left internal carotid artery, right vertebral artery, left vertebral 
artery 
E. Right internal carotid artery, left internal carotid artery, right posterior cerebral artery, left 
posterior cerebral artery 
 
Answer: D. Right internal carotid artery, left internal carotid artery, right vertebral artery, left vertebral 
artery 
 
Rationale: The 4 major blood vessels coursing through the neck to supply blood to the brain are the 
right internal carotid artery, left internal carotid artery, right vertebral artery, and left vertebral artery. 
There are no such arteries as right and left cerebral artery. The external carotid arteries supply other 
structures of the head and neck. The anterior and posterior cerebral arteries are intracranial arteries 
that do not course through the neck. 
 
Reference:  
1.) Maas MB, Safdieh JE. Ischemic stroke: pathophysiology and principles of localization. Neurology. 
2009;13(Part 1):2‐16. 
 
 
14. Question: A 34‐year‐old man developed acute onset of severe headache followed by emesis and loss 
of consciousness. He was taken to an outside emergency department where head CT revealed a diffuse, 
thick subarachnoid hemorrhage with intraventricular hemorrhage. CT angiogram demonstrated a basilar 
tip aneurysm. After transfer to a different hospital, he underwent successful coiling of the aneurysm and 
placement of an external ventricular drain for hydrocephalus. Current Glasgow Coma Scale score is 10T 
(eye opening 3, verbal response 1T, motor response 6). Which of the following therapies is most likely to 
improve outcomes in delayed cerebral ischemia? 
 
A. Fluid boluses to maintain positive fluid balance 
B. Hypertonic saline to ensure sodium level greater than 140 mEq/L 
C. Hypervolemic, hypertensive, and hemodilutional therapy 
D. Nimodipine, 60 mg every 6 hours  
E. Vasopressor infusion to maintain systolic blood pressure of greater than 180 mm Hg 
 
Answer: D. Nimodipine, 60 mg every 6 hours 
 
Rationale: Nimodipine, a calcium channel blocker, is recommended by both American Heart Association 
and Neurocritical Care Society subarachnoid hemorrhage (SAH) guidelines to be administered to all 
patients with SAH for 21 days. Studies have demonstrated class I evidence that nimodipine 

 
10
Neurocritical Care Review 
Practice Questions 
Section 1. 
 
administration in patients with SAH results in better functional outcomes. It did not demonstrate a 
reduction in angiographic vasospasm. If a patient with SAH does develop symptomatic cerebral 
vasospasm and delayed cerebral ischemia, it is reasonable to initiate a trial of induced hypertension if 
the aneurysm is secured, with the end goal of resolution of deficits. Norepinephrine and phenylephrine 
are the preferred agents since the brain lacks α1 receptors. Norepinephrine and phenylephrine result in 
systemic vasoconstriction and not cerebral vasoconstriction. Hypervolemic, hypertensive, and 
hemodilutional therapy have been shown to result in adverse outcomes dues to hemodilution. 
Guidelines recommend targeting euvolemia and recommend against the use of prophylactic 
hypervolemia. There is no known association between higher sodium goals and better outcomes with 
SAH. Guidelines recommend fludrocortisone and hypertonic saline only for correction of hyponatremia. 
 
References:  
1.) Connolly ES Jr, Rabinstein AA, Carhuapoma JR, et al; American Heart Association Stroke Council; 
Council on Cardiovascular Radiology and Intervention; Council on Cardiovascular Nursing; Council on 
Cardiovascular Surgery and Anesthesia; Council on Clinical Cardiology. Guidelines for the management 
of aneurysmal subarachnoid hemorrhage: a guideline for healthcare professionals from the American 
Heart Association/American Stroke Association. Stroke. 2012 Jun;43(6):1711‐1737. 
 
2.) Diringer MN, Bleck TP, Hemphill JC 3rd, et al; Neurocritical Care Society. Critical care management of 
patients following aneurysmal subarachnoid hemorrhage: recommendations from the Neurocritical Care 
Society's Multidisciplinary Consensus Conference. Neurocrit Care. 2011 Sep;15(2):211‐240. 
 
 
15. Question: A 48‐year‐old woman with a history of hypertension, diabetes mellitus, and 
hyperlipidemia presents with a 1‐hour history of aphasia, right‐sided weakness, and altered mentation. 
Glasgow Coma Scale score is 10. She is admitted to the neuro‐ICU with a left frontal lobar hemorrhage 
with a left‐to‐right midline shift of 2 mm without intraventricular extension or hydrocephalus. Repeat 
head CT 6 hours later shows the stable size of the hematoma without hematoma expansion. 
Neurosurgery obtains the family’s consent for minimally invasive, image‐guided hematoma evacuation 
and repeated doses of tissue plasminogen activator to be administered via surgical drain that will 
remain within the hematoma postoperatively. While the patient is in surgery, the family asks for a 
review of the benefits of the procedure. The most appropriate response is that the intervention 
 
A. decreases all‐cause mortality and improves functional outcomes at 1 year 
B. decreases all‐cause mortality and may improve functional outcomes at 1 year if a threshold 
volume of clot is removed 
C. increases all‐cause mortality but will improve functional outcomes at 1 year in those who 
survive 
D. decreases all‐cause mortality but results in worse functional outcomes at 1 year 
 
Answer: B. decreases all‐cause mortality and may improve functional outcomes at 1 year if a threshold 
volume of clot is removed 
 
Rationale: For spontaneous intracerebral brain hemorrhage volume greater than 30 mL in patients with 
a baseline Glasgow Coma Scale score less than 14, minimally invasive, image‐guided, hematoma 

 
11
Neurocritical Care Review 
Practice Questions 
Section 1. 
 
evacuation followed by repeat doses of intracatheter tissue plasminogen activator showed a benefit in 
all‐cause mortality when compared with those receiving standard medical management. Additionally, in 
patients with a post‐evacuation hematoma volume of less than 15 mL, there is an increased chance of 
good outcomes at 1 year. 
 
Reference:  
1.) Hanley DF, Thompson RE, Rosenblum M, et al; MISTIE III Investigators. Efficacy and safety of 
minimally invasive surgery with thrombolysis in intracerebral haemorrhage evacuation (MISTIE III): a 
randomised, controlled, open‐label, blinded endpoint phase 3 trial. Lancet. 2019 Mar 
9;393(10175):1021‐1032. 
 
 
16. Question: A 63‐year‐old woman presents to the emergency department 
with right‐sided weakness and aphasia that started 5 hours ago. She has 
global aphasia and is not following commands, with an NIH Stroke Scale 
score of 15. Glucose is 120 mg/dL. No seizure‐like activity is reported. Head 
CT is normal. Head and neck CT angiogram is shown below. Vital signs are: 
temperature 36.8 °C (98.2°F), blood pressure 170/85 mm Hg, heart rate 98 
beats/min, respiratory rate 18 breaths/min, and oxygen saturation 93% on 
2 L nasal cannula. Which of the following is the most appropriate next 
intervention? 
 
A. IV thrombolytic administration 
B. Admission to neuro‐ICU for monitoring with neurosurgery consultation for possible 
decompressive hemicraniectomy. 
C. Neuroendovascular consultation for emergent endovascular thrombectomy 
D. Continuous EEG monitoring 
E. CT perfusion 
 
Answer: C. Neuroendovascular consultation for emergent endovascular thrombectomy 
 
Rationale: The CT angiogram reveals a left middle cerebral artery occlusion. This patient is outside the 
window for IV thrombolysis but is a candidate for mechanical thrombectomy. Eligibility criteria for 
mechanical thrombectomy in acute stroke are:  
1. Pre‐stroke modified Rankin Scale score of 0‐1 
2. Causative occlusion of the internal carotid artery or middle cerebral artery segment 1 (M1) 
3. Age 18 years or older 
4. NIH Stroke Scale score of 6 or greater 
5. Ability to initiate treatment (groin puncture) within 6 hours of symptom onset 
 
She may need a decompressive hemicraniectomy if she develops malignant cerebral edema but must 
first undergo emergent treatment for the occlusion. Continuous EEG may be warranted in the future but 
not at this point because there is a lack of findings concerning for seizure activity and this would delay 
emergent treatment. Emergent CT perfusion is not needed at this point because she is within the 
thrombectomy window. 

 
12
Neurocritical Care Review 
Practice Questions 
Section 1. 
 
 
Reference: 
1.) Warner JJ, Harrington RA, Sacco RL, Elkind MSV. Guidelines for the early management of patients 
with acute ischemic stroke: 2019 update to the 2018 guidelines for the early management of acute 
ischemic stroke. Stroke. 2019 Dec;50(12):3331‐3332. 
 
 
Question 17: A 23‐year‐old male police officer was assaulted 8 hours ago. Head CT is normal. Neck CT 
demonstrates a C5‐C6 transverse foramen fracture on the left. On initial examination, cranial nerves are 
intact, and he follows commands in all 4 extremities with full strength. One hour later, he has irregular 
breathing, a large and unreactive left pupil, absent left corneal reflex, dysconjugate gaze, and no 
movement of the right upper and lower extremities. Emergent CT angiogram of the head and neck 
reveals a left vertebral artery dissection as a basilar artery occlusion. Which of the following is the most 
appropriate next intervention?  
 
A. Aspirin administration 
B. IV thrombolysis administration 
C. Neuroendovascular consultation for evaluation of emergent thrombectomy 
D. Continuous EEG 
E. Emergent CT perfusion 
 
Answer: C. Neuroendovascular consultation for evaluation of emergent thrombectomy 
 
Rationale: Arterial dissection as the etiology of stroke occurs in about 25% of patients aged 45 years and 
younger. Up to 10% of extracranial traumatic vertebral artery dissections can extend intracranially. In 
this patient, a vertebral artery dissection progressed into an ischemic stroke due to a thrombus. Because 
of the potential for severe disability and mortality from a basilar artery occlusion, mechanical 
thrombectomy should be considered and the neurointerventional team should be consulted 
emergently. Aspirin is an appropriate therapy for secondary stroke prevention in the case of dissection, 
but this patient has an acute ischemic event due to occlusion and needs emergent reperfusion. Because 
of the recent trauma and symptom onset, he is not a candidate for IV thrombolytics. Continuous EEG 
can be considered for persistent alteration of mental status; however, his current symptomatology 
makes it less likely that a seizure is the etiology of the decline. CT perfusion is not indicated since a clear 
occlusion is seen on CT angiogram with associated neurologic deficits. He should undergo emergent 
thrombectomy. 
 
References:  
1.) Fields JD, Lutsep HL, Rymer MR, et al; Merci Registry Investigators. Endovascular mechanical 
thrombectomy for the treatment of acute ischemic stroke due to arterial dissection. Interv Neuroradiol. 
2012 Mar;18(1):74‐79.  
 
2.) Simon LV, Nassar Ak, Mohseni M. Vertebral artery injury. In: StatPearls [Internet]. Treasure Island 
(FL): StatPearls Publishing; 2021 Jan. 2020 Jul 21. 
 
 

 
13
Neurocritical Care Review 
Practice Questions 
Section 1. 
 
Question 18: A 36‐year‐old man with a history of IV drug abuse is brought to the emergency department 
for unresponsiveness by emergency medical services, who intubated him on the scene for airway 
protection. Vital signs are: temperature 39.1 °C (102.3 °F), heart rate 136 beats/min, blood pressure 
134/64 mm Hg, oxygen saturation 93% on mechanical ventilation with 100% Fio2 and positive end‐
expiratory pressure 12 cm H2O. While off sedation he is able to open eyes, track the examiner, follow 
simple commands (wiggle toes, thumbs up), and has symmetric purposeful movements. Antibiotics are 
started for an opacity on chest radiograph and he remains on sedation for ventilator synchrony. Because 
of continuous sedation, neurologic examination is limited to pupillary examination. Blood cultures show 
gram‐positive cocci. Echocardiography reveal a 3‐mm vegetation in the mitral valve. On admission day 3, 
a nurse notices a pupillary change: 5‐mm fixed right pupil and 3‐mm and reactive left pupil. Blood 
pressure is 180/95 mm Hg and heart rate 45 beats/min. Most recent sodium is 134 mEq/L. Emergent 
head CT shows a large right cerebral hemispheric infarct with uncal herniation. Which of the following 
is/are the most appropriate next step(s)? 
 
A. Administer 30 mL 23.4% saline and request emergent neurosurgical consultation. 
B. Administer mannitol, 1 mg/kg, and request emergent neurosurgical consultation. 
C. Update the family on the patient’s status. 
D. Obtain emergent CT angiogram of brain and neck to evaluate for large vessel occlusion. 
E. Administer IV alteplase, 0.9 mg/kg. 
 
Answer: A. Administer 30 mL 23.4% saline and request emergent neurosurgical consultation. 
 
Rationale: This patient has herniation from a large right cerebral infarction with malignant brain edema. 
The immediate goal is to reduce intracranial pressure with hyperosmolar therapy and consult 
neurosurgery for consideration of decompression. Although mannitol can be used as hyperosmolar 
therapy, the dose required is 1 g/kg, not 1 mg/kg. Because he has an established infarct on CT and last 
known normal was at time of presentation 3 days ago, alteplase is not indicated. Although it is 
important to update the family on the patient’s status, acute management of brain herniation is the 
priority. 
 
Reference: 
1.) Cook AM, Jones GM, Hawryluk GWJ, et al. Guidelines for the acute treatment of cerebral edema in 
neurocritical care patients. Neurocrit Care. 2020 Jun;32(3):647‐666. 
 
 
Question 19: Which of the following statements is true regarding risk factors for subarachnoid 
hemorrhage (SAH)? 
 
A. Aneurysms are more likely to be detected in children of patients with SAH than in siblings of 
patients with SAH. 
B. Genetic and familial risk factors are more important than environmental risk factors for SAH. 
C. Hypertension, alcohol intake, and smoking are modifiable risk factors for SAH. 
D. SAH is more common in Caucasian populations than Hispanic populations. 
E. SAH is more common in men than in women. 
 

 
14
Neurocritical Care Review 
Practice Questions 
Section 1. 
 
Answer: C. Hypertension, alcohol intake, and smoking are modifiable risk factors for SAH. 
 
Rationale: Subarachnoid hemorrhage (SAH) is more common in African American and Hispanic 
populations than in Caucasian populations. SAH is more common in women (female‐to‐male ratio 1.6:1). 
Aneurysms are more likely to be detected in siblings of patients with SAH than children of patients with 
SAH. Several studies, including a twin study, demonstrated that environmental risk factors are more 
important than genetic or familial inheritance with regard to aneurysm and SAH risk. 
 
References:  
1.) Macdonald RL, Schweizer TA. Spontaneous subarachnoid haemorrhage. Lancet. 2017 Feb 
11;389(10069):655‐666. 
 
2.) Brown RD Jr, Broderick JP. Unruptured intracranial aneurysms: epidemiology, natural history, 
management options, and familial screening. Lancet Neurol. 2014 Apr;13(4):393‐404. 
 
3.) Muehlschlegel S. Subarachnoid hemorrhage. Continuum (Minneap Minn). 2018 Dec;24(6):1623‐1657. 
 
 
 
 
 
 

 
15
Neurocritical Care Review 
Practice Questions 
Section 2. 
 
 

Question 1: A 78‐kg (172‐lb), 57‐year‐old man with no known past medical history presents to the 
emergency department (ED) after the family called emergency medical services (EMS) for abnormal 
behavior. EMS reports ongoing generalized tonic‐clonic seizure activity that began 17 minutes ago. EMS 
administered 4 mg of IV lorazepam. When the patient arrives in the ED, he is in generalized convulsive 
status epilepticus and is hypoxic and tachycardic. Fingerstick glucose is 278 mg/dL. To secure his airway, 
he is intubated with rocuronium and propofol. The most appropriate next step is to administer 
 
A. IV lacosamide, 400 mg 
B. IV levetiracetam, 60 mg/kg 
C. IV levetiracetam, 1500 mg 
D. a second dose of IV lorazepam, 4 mg  
E. IV thiamine 
 
Answer: B. IV levetiracetam, 60 mg/kg 
 
Rationale: This patient has received an adequate dose of a benzodiazepine as recommended by 
American Epilepsy Society (AES) and Neurocritical Care Society (NCS) guidelines. The recommended 
dose of IV lorazepam is 0.1 mg/kg, with a maximum of 4 mg. Since he has had generalized seizure 
activity for 16 minutes refractory to a benzodiazepine, he is in established status epilepticus. The next 
step is to administer IV levetiracetam, 60 mg/kg, which was found to be equally efficacious as 
fosphenytoin, 20 mg/kg, and valproate, 40 mg/kg, in the landmark ESETT trial. A dose of 1500 mg 
levetiracetam IV for a 78‐kg (172‐lb) man would be too low. Lacosamide is not an option for urgent 
control therapy (also known as second therapy phase) for convulsive status epilepticus based on current 
AES and NCS guidelines. IV thiamine seems unnecessary at this time since there is no evidence of alcohol 
abuse. A second dose of lorazepam could be considered based on guideline recommendations but since 
he has already been intubated with propofol (another GABAergic medication), it is probably more 
beneficial to initiate urgent control therapy at this time. 
 
References:  
1.) Brophy GM, Bell R, Claassen J, et al; Neurocritical Care Society Status Epilepticus Guideline Writing 
Committee. Guidelines for the evaluation and management of status epilepticus. Neurocrit Care. 2012 
Aug;17(1):3‐23. 
 
2.). Glauser T, Shinnar S, Gloss D, et al. Evidence‐based guideline: treatment of convulsive status 
epilepticus in children and adults: report of the Guideline Committee of the American Epilepsy Society. 
Epilepsy Curr. 2016 Jan‐Feb;16(1):48‐61. 
 
 
 
 
 
 
 

 
16
Neurocritical Care Review 
Practice Questions 
Section 2. 
 
Question 2: Studies have demonstrated that corticosteroids worsen outcomes in patients with which of 
the following diagnoses? 
 
A. Cerebral neoplasm, anoxic brain injury, and traumatic brain injury 
B. Intracerebral hemorrhage, cerebral infarction, and anoxic brain injury 
C. Traumatic brain injury, cerebral infarction, and anoxic brain injury 
D. Traumatic brain injury, cerebral neoplasm, and intracerebral hemorrhage  
E. Traumatic brain injury, intracerebral hemorrhage, and cerebral infarction 
 
Answer: E. Traumatic brain injury, intracerebral hemorrhage, and cerebral infarction 
 
Rationale: The American Heart Association states that the routine use of dexamethasone in patients 
with acute ischemic stroke is not recommended. The CRASH trial demonstrated an increased mortality 
in patients with traumatic brain injury treated with 48 hours of methylprednisolone compared with 
placebo. Additionally, the routine use of corticosteroids in patients with intracerebral brain hemorrhage 
is not recommended. 
 
References:  
1.) Wijdicks EF, Sheth KN, Carter BS, et al; American Heart Association Stroke Council. Recommendations 
for the management of cerebral and cerebellar infarction with swelling: a statement for healthcare 
professionals from the American Heart Association/American Stroke Association. Stroke. 2014 
Apr;45(4):1222‐1238. 
 
2.) Hemphill JC 3rd, Greenberg SM, Anderson CS, et al; American Heart Association Stroke Council; 
Council on Cardiovascular and Stroke Nursing; Council on Clinical Cardiology. Guidelines for the 
management of spontaneous intracerebral hemorrhage: a guideline for healthcare professionals from 
the American Heart Association/American Stroke Association. Stroke. 2015 Jul;46(7):2032‐2060. 
 
3.) Roberts I, Yates D, Sandercock P, et al; CRASH trial collaborators. Effect of intravenous corticosteroids 
on death within 14 days in 10008 adults with clinically significant head injury (MRC CRASH trial): 
randomised placebo‐controlled trial. Lancet. 2004 Oct 9‐15;364(9442):1321‐1328. 
 
 
Question 3: Which of the following statements regarding cerebral edema and intracranial pressure (ICP) 
elevation is true? 
 
A. Cerebral edema does not affect intracranial volume. 
B. Cytotoxic edema typically affects white matter only. 
C. Focal cerebral edema can result in cerebral herniation with or without ICP elevation. 
D. The most common cause of cytotoxic edema is neoplasm. 
E. Traumatic brain injury preferentially results in vasogenic edema. 
 
Answer: C. Focal cerebral edema can result in cerebral herniation with or without ICP elevation. 
 

 
17
Neurocritical Care Review 
Practice Questions 
Section 2. 
 
Rationale: Patients may develop herniation syndromes in the setting of normal intracranial pressure 
(ICP). Global cerebral edema typically results in a global rise in ICP. Cerebral edema increases intracranial 
volume. Cytotoxic edema affects both gray and white matter. The most common cause of cytotoxic 
edema is ischemic stroke. Traumatic brain injury results in both cytotoxic and vasogenic edema. 
 
Reference: 
1.) Koenig MA. Cerebral edema and elevated intracranial pressure. Continuum (Minneap Minn). 2018 
Dec;24(6):1588‐1602. 
 
 
Question 4: A 56‐year‐old man is admitted to the neuro‐ICU with a hemorrhagic stroke. He was 
intubated for airway protection. One week into his hospital stay, the respiratory therapist notices an 
acute rise in end‐tidal CO2. Which of the following is the most appropriate next intervention? 
 
A. Albuterol inhaler 
B. Chest compressions 
C. Suction 
D. Heparin infusion 
 
Answer: C. Suction 
 
Rationale: A rise in end‐tidal CO2 (ETCO2) can be due to increased CO2 production or decreased alveolar 
ventilation. This can be seen in an obstruction from secretions, mucous plugging requiring suctioning, or 
oversedation. Decreased ETCO2 is seen in respiratory depression or acute bronchoconstriction. An 
albuterol inhaler would be needed in acute bronchoconstriction. Chest compressions would be needed 
in cardiac arrest where there a flattened waveform unless chest compressions are occurring. A heparin 
infusion would be needed for a pulmonary embolism, in which case a low ETCO2 could show due to 
shunt. 
 
References:  
1.) Kodali BS. Capnography outside the operating rooms. Anesthesiology. 2013 Jan;118(1):192‐201. 
 
 
Question 5: A previously healthy 27‐year‐old man is admitted with a traumatic cervical spinal cord injury 
after diving into a shallow pool. He was intubated on arrival in the emergency department because of 
quadriplegia and respiratory distress. Six days into his hospital course he develops hypoxia. Chest 
radiograph shows bilateral opacities. Ventilator settings are: pressure‐regulated volume control, tidal 
volume 450 mL, respiratory rate 20 breaths/min, peak end‐expiratory pressure 10 cm H2O, Fio2 80%, 
with the following arterial blood gas analysis results: pH 7.34/ Paco2 44 mm Hg/ Pao2 62 mm Hg/HCO3 23 
mEq/L. Which of the following is a treatment proven to benefit outcomes for his condition? 
 
A. Glucocorticoids 
B. Inhaled nitric oxide 
C. Pressure‐regulated volume control ventilation 
D. Prone positioning 

 
18
Neurocritical Care Review 
Practice Questions 
Section 2. 
 
 
Answer: D. Prone positioning 
 
Rationale: This patient has severe acute respiratory distress syndrome (ARDS) as defined by the Berlin 
definition: 1) respiratory symptoms within 1 week of clinical insult, 2) bilateral opacities, 3) condition not 
explained by cardiac failure or fluid overload, and 4) hypoxia defined by the P/F ratio as determined on 
peak end‐expiratory pressure (PEEP) of 5 cm H2O or greater. The severity of ARDS is defined as mild (P/F 
ratio 201‐300), moderate (P/F ratio 101‐200), or severe (P/F ratio < 100). Of the options, only prone 
positioning has been shown to improve outcomes. Treatment for ARDS is largely supportive and is 
aimed at lung protection strategies of ventilation, treating the underlying cause of the ARDS, and 
improving oxygenation. Prone positioning increases dorsal alveoli recruitment, increasing lung 
compliance. By optimizing patient selection and treatment protocols, the Proning Severe ARDS Patients 
(PROSEVA) trial demonstrated a significant mortality benefit with prone positioning. Other treatments 
that show an outcome benefit include optimizing PEEP with esophageal balloon monitoring, decreasing 
oxygen demand with sedation and analgesics, neuromuscular blockade to avoid ventilator asynchrony 
and chest wall rigidity, and conservative fluid management that leads to increased ventilator‐ and ICU‐
free days. There are inconsistent data for the benefit of glucocorticoids in ARDS; overall, it seems that 
they show no mortality benefit. Inhaled nitric oxide improves oxygenation but has not been shown to 
improve morbidity or mortality. Airway pressure release ventilator mode has been found to reduce 
mortality, not pressure‐regulated volume control mode. 
 
References:  
1.) ARDS Definition Task Force; Ranieri VM, Rubenfeld, et al. Acute respiratory distress syndrome: the 
Berlin Definition. JAMA. 2012 Jun 20;307(23):2526‐2533. 
 
2.) Papazian L, Aubron C, Brochard L, et al. Formal guidelines: management of acute respiratory distress 
syndrome. Ann. Intensive Care. 2019 Jun 13;9(1):69. 
 
 
Question 6: Pulse pressure variation is 
 
A. a static measure of preload 
B. elevated in decompensated heart failure 
C. elevated in hypovolemia 
D. low in hemorrhagic shock 
 
Answer: C. elevated in hypovolemia 
 
Rationale: Pulse pressure variation (PPV) is a dynamic measure of preload used in assessing fluid 
responsiveness in mechanically ventilated patients. It quantifies the changes in arterial pulse pressure 
during mechanical ventilation. Pulse pressure is the difference between systolic and diastolic pressure. 
PPV is the difference between maximum and minimum pulse pressure over one mechanical breath. 
During inspiration, the increased intrathoracic pressure causes decreased venous return to the right 
ventricle while left ventricular filling is increased, leading to an increase in left ventricular stroke volume. 
During expiration, left ventricular stroke volume is decreased due to reduced right ventricle filling during 

 
19
Neurocritical Care Review 
Practice Questions 
Section 2. 
 
inspiration. Changes are marked in a hypovolemic patient, leading to increased PPV greater than 13%. 
PPV is a dynamic, not static, measurement. Decompensated heart failure leads to increased right 
ventricular pressures and low PPV. Hemorrhagic shock shows an elevated PPV, indicating fluid 
responsiveness. 
 
References:  
1.) Teboul JL, Monnet X, Chemla D, Michard F. Arterial pulse pressure variation with mechanical 
ventilation. Am J Respir Crit Care Med. 2019 Jan 1;199(1):22‐31. 
 
 
Question 7: A 66‐year‐old man with a history of hypertension and type 2 diabetes is admitted to the ICU 
after unsuccessful mechanical thrombectomy of the left middle cerebral artery. He suddenly becomes 
tachycardic, and systolic blood pressure drops below the goal range of less than 140 mm Hg. EEG is 
shown below. Vital signs are: temperature 37 °C (99 °F), respiratory rate 14 breaths/min, Spo2 greater 
than 94%, blood pressure 132/72 mm Hg, heart rate 142 beats/min. Laboratory tests are significant for 
serum electrolytes of sodium 138 mEq/L, potassium 3.2 mEq/L, BUN 30 mg/dL, creatinine 1.3 mg/dL, 
and magnesium 1.0 mg/dL. Which of the following is the most appropriate course of action? 

 
 
A. No treatment is indicated since the patient is hemodynamically stable and there is a high 
incidence of spontaneous cardioversion in critically ill patients. 
B. Administer replete magnesium, which has been shown to be equally effective as amiodarone in 
obtaining rate control. 
C. Arrange for immediate synchronized cardioversion using 150J because of its effectiveness in 
rhythm control and low relapse rate. 
D. Start amiodarone, which is superior to magnesium in efficacy for rate control. 
E. Start norepinephrine to achieve blood pressure goal. 
 
Answer: B. Administer replete magnesium, which has been shown to be equally effective as amiodarone 
in obtaining rate control. 
 
Rationale: This patient is not within the target blood pressure goal. Spontaneous cardioversion occurs in 
only about two‐thirds of patients in the inpatient setting. Electrocardioversion of atrial fibrillation (AF) in 
critically ill patients is less likely to be successful; most will revert to AF. Magnesium level is low; 
repletion of magnesium has been shown to be equally effective as amiodarone in obtaining rate control. 
Norepinephrine will not address the AF. 
 
References: 

 
20
Neurocritical Care Review 
Practice Questions 
Section 2. 
 
1.) Moran JL, Gallagher J, Peake SL, Cunningham DN, Salagaras M, Leppard P. Parenteral magnesium 
sulfate versus amiodarone in the therapy of atrial tachyarrhythmias: a prospective, randomized 
study. Crit Care Med. 1995 Nov;23(11):1816‐1824. 
 
2.) Mayr A, Ritsch N, Knotzer H, et al. Effectiveness of direct‐current cardioversion for treatment of 
supraventricular tachyarrhythmias, in particular atrial fibrillation, in surgical intensive care patients. Crit 
Care Med. 2003 Feb;31(2):401‐405. 
 
3.) Danias PG, Caulfield TA, Weigner MJ, Silverman DI, Manning WJ. Likelihood of spontaneous 
conversion of atrial fibrillation to sinus rhythm. J Am Coll Card. 1998 Mar 1;31(3)588‐592. 
 
 
Question 8: A 37‐year‐old woman is mechanically 
ventilated in the ICU on postoperative day 1 after a 
complex stenting procedure for an unruptured aneurysm. 
She is on a heparin infusion for in‐stent clot formation 
and propofol, 75 µg/kg/min. Ventilation settings are: 
volume control 8 mL/kg ideal body weight, peak end‐
expiratory pressure 5 cm H2O, respiratory rate 14 
breaths/min, Fio2 35%, and Spo2 greater than 96%. 
Morning laboratory results show anti‐Xa 0.4 IU/mL, 
hemoglobin 9.2 g/dL, sodium 140 mEq/L, and creatinine 0.8 mg/dL. Blood pressure is 138/65 mm Hg 
and heart rate 80 beats/min. She has a sudden drop in blood pressure (90/78 mm Hg) and heart rate 
accelerates to 122 beats/min. Repeat laboratory results show hemoglobin 7.8 g/dL, lactate 4.5 mmol/L, 
creatinine 1.1 mg/dL, anti‐Xa 0.6 IU/mL, and pH 7.21. Radial arterial line waveform is shown below, with 
pulse pressure variation 14%. If a pulmonary artery catheter were used, what would be the most likely 
changes in measurements, where CVP is central venous pressure, PAOP is pulmonary artery occlusion 
pressure, CO is cardiac output, and SVR is systemic vascular resistance? 
 
A. CVP↑ PAOP↑ CO↓ SVR↑ 
B. CVP↓ PAOP↓ CO↓ SVR↑ 
C. CVP↓ PAOP↓ CO↑ SVR↓ 
D. CVP↕ PAOP↓ CO↑ SVR↑ 
 
Answer: B. CVP↓ PAOP↓ CO↓ SVR↑ 
 
Rationale: This patient is in shock. The greater‐than‐12% pulse pressure variation also suggests volume 
responsiveness. She is on a heparin infusion with a drop in hemoglobin, which should imply acute blood 
loss as the cause of shock. This would result in low central venous pressure, pulmonary artery occlusion 
pressure, cardiac output, and high systemic vascular resistance. 
 
References:  
1.) Yang X, Du B. Does pulse pressure variation predict fluid responsiveness in critically ill patients? A 
systematic review and meta‐analysis. Crit Care. 2014 Nov 27;18(6):650. 
 

 
21
Neurocritical Care Review 
Practice Questions 
Section 2. 
 
Question 9: A 56‐year‐old man with a previous 
medical history of hypertension and diabetes 
presents with acute onset of nausea, vomiting, 
vertigo, and gait instability. He is admitted to the 
ward and treated for gastroenteritis. The next 
morning, a nurse finds him obtunded with 
bilateral dilated and sluggish pupils. Noncontrast 
head CT reveals a right cerebellar hypodensity 
with associated edema, effacement of the fourth 
ventricle, and ventriculomegaly in the lateral and 
third ventricles, as shown below. The nurse reports that he has 2 peripheral IVs and his vital signs are 
stable. He localizes to pain in bilateral upper extremities. Which of the following is/are the next step(s) 
in management? 
 
A. Emergent 23% sodium chloride, 30 mL IV bolus 
B. Mannitol bolus, 1 gm/kg IV 
C. Mannitol, 1 gm/kg IV, and neurosurgery consult for external ventricular drain (EVD) placement 
and decompressive craniectomy 
D. Placement of central line, emergent 23% sodium chloride, 30 mL IV bolus, and neurosurgery 
consult for EVD placement 
E. Neurosurgery consult for EVD placement only 
 
Answer: C. Mannitol, 1 gm/kg IV, and neurosurgery consult for external ventricular drain (EVD) 
placement and decompressive craniectomy 
 
Rationale: Although the evidence for the efficacy of hyperosmolar therapy in acute ischemic stroke is 
weak, mannitol and hypertonic saline are often used as temporizing measures. In this situation mannitol 
is preferred over 23% sodium chloride, since this patient has only peripheral IVs and there would be an 
unacceptable delay in administration of hyperosmolar therapy if time were taken to secure central 
venous access. Since hyperosmolar therapy is not a definitive treatment, it must be followed by 
decompressive craniectomy. External ventricular drain (EVD) placement alone would likely lead to 
upward cerebellar herniation. American Heart Association guidelines recommend against EVD 
placement as the primary intervention for obstructive hydrocephalus from posterior fossa strokes. 
 
References:  
1.) Wijdicks EF, Sheth KN, Carter BS, et al; American Heart Association Stroke Council. Recommendations 
for the management of cerebral and cerebellar infarction with swelling: a statement for healthcare 
professionals from the American Heart Association/American Stroke Association. Stroke. 2014 
Apr;45(4):1222‐1238. 
 
2.) Misra UK, Kalita J, Ranjan P, Mandal SK. Mannitol in intracerebral hemorrhage: a randomized 
controlled study. J Neurol Sci. 2005 Jul 15;234(1‐2):41‐45. 
 
3.) Kase CS, Wolf PA. Cerebellar infarction: upward transtentorial herniation after ventriculostomy. 
Stroke. 1993 Jul;24(7):1096‐1098. 

 
22
Neurocritical Care Review 
Practice Questions 
Section 2. 
 
Question 10: A 21‐year‐old man with no medical history is admitted to the neuro‐ICU with a traumatic 
brain injury following a head‐on motor vehicle collision. Neuroimaging reveals bilateral scattered 
traumatic subarachnoid hemorrhage, small cerebral contusions, and diffuse axonal injury. On ICU 
admission, Glasgow Coma Scale score is 8T. Neurosurgery is consulted, and an external ventricular drain 
(EVD) is placed. He is mechanically ventilated and receiving fentanyl and propofol infusions for agitation 
and tachycardia. Current vital signs are: temperature 37.9 °C (100.2 °F), heart rate 111 beats/min, 
respiratory rate 14 breaths/min, blood pressure 149/83 mm Hg, and oxygen saturation 99% on Fio2 50%. 
Morning laboratory test results are: WBC 12,200/µL, hemoglobin 10.2 g/dL, platelets 302,000/µL, 
sodium 137 mEq/L, creatinine 0.9 mg/dL, and glucose 107 mg/dL. Arterial blood gas analysis reveals pH 
7.27, Paco2 50 mm Hg, Pao2 86 mm Hg, HCO3 25 mEq/L. Intracranial pressure has been 27 mm Hg for 
the past 15 minutes. Which of the following is the most appropriate next step? 
 
A. Administer 23% sodium chloride 30 mL IV bolus. 
B. Increase ventilator set respiratory rate. 
C. Initiate induced hypothermia. 
D. Administer mannitol, 1 gm/kg IV. 
E. Administer pentobarbital, 20 mg/kg IV bolus. 
 
Answer: B. Increase ventilator set respiratory rate. 
 
Rationale: First‐line treatment for elevated intracranial pressure (ICP) consists of noninvasive 
maneuvers, which include optimization of sedation and analgesia and normalizing temperature and 
CO2. In this patient, arterial blood gas analysis reveals an acute respiratory acidosis. Acidosis results in 
cerebral vasodilation, thereby increasing ICP. Increasing the set respiratory rate on the ventilator should 
lower the Paco2 to normal range and potentially lower ICP. If first‐line noninvasive measures fail to 
reduce ICP to less than 22 mm Hg, the next‐line therapy is hyperosmolar therapy with hypertonic saline 
or mannitol and cerebrospinal fluid drainage. If intracranial pressures continue to be refractory, 
pentobarbital could be considered. 
 
References:  
1.) Cadena R, Shoykhet M, Ratcliff JJ. Emergency neurological life support: intracranial hypertension and 
herniation. Neurocrit Care. 2017 Sep;27(Suppl 1):82‐88. 
 
 
Question 11: Which of the following statements is true regarding cerebral herniation syndromes? 
 
A. Intracranial pressure elevations typically precede pupillary dilation in transtentorial cerebral 
herniation. 
B. Pupillary dilation is uncommon with cerebellar upward herniation and tonsillar herniation. 
C. Transtentorial herniation is universally fatal. 
D. A sequela of transtentorial herniation called duret hemorrhage is defined as a posterior cerebral 
artery (PCA) stroke due to compression of the contralateral PCA against the tentorial edge.  
E. With transtentorial herniation, pupil dilation is contralateral to the lesion. 
 
Answer: B. Pupillary dilation is uncommon with cerebellar upward herniation and tonsillar herniation. 

 
23
Neurocritical Care Review 
Practice Questions 
Section 2. 
 
 
Rationale: Since cerebellar upward herniation and tonsillar herniation often do not manifest with pupil 
dilation, this herniation syndrome is more likely to be missed or delayed. Intracranial pressure 
elevations are typically a late finding while pupillary dilation is an early finding in transtentorial 
herniation. With transtentorial herniation, hemiparesis can be contralateral or ipsilateral to the lesion 
due to direct corticospinal tract involvement or compression of the contralateral cerebral peduncle 
against the tentorial edge (Kernohan notch). Depending on the cause of herniation and the availability 
of medical and surgical treatment, transtentorial herniation may be reversible and patients may be able 
to regain independence. A duret hemorrhage is a pontine hemorrhage that results from compression of 
venous drainage leading to venous congestion and hemorrhagic infarction in medial midbrain and 
pontine structures. With transtentorial herniation, pupil dilation is ipsilateral to the lesion. 
 
References: 
1.) Koenig MA. Cerebral edema and elevated intracranial pressure. Continuum (Minneap Minn). 2018 
Dec;24(6):1588‐1602. 
 
 
Question 12: Neurogenic pulmonary edema is due to 
 
A. catecholamine surge 
B. depressed ejection fraction 
C. gastric contents from aspiration 
D. inflammation 
 
Answer: A. catecholamine surge 
 
Rationale: The onset of neurogenic pulmonary edema is typically at the onset of the neurologic insult 
and occurs in about 40% of all neurology patients with a higher incidence in patients with intracerebral 
hemorrhage. Pulmonary edema of acute respiratory distress syndrome/acute lung injury is different and 
is due to an inflammatory reaction to lung injury and the proteinaceous fluid is from the pneumocytes 
within the alveolar wall. Pulmonary edema from a depressed ejection fraction is seen in heart failure. 
Aspiration leads to aspiration pneumonitis. 
 
References:  
1.) Busl KM and Bleck TP. Neurogenic pulmonary edema. Crit Care Med. 2015 Aug;43(8):1710‐1715. 
 
 
 
 
 
 
 
 

 
24
Neurocritical Care Review 
Practice Questions 
Section 2. 
 
Question 13: A 51‐year‐old woman is postoperative day 1 after a T10‐L3 laminectomy. She has new‐
onset tachycardia, and her physician is preparing to administer adenosine. Which of the following is 
false about adenosine? 
 
A. It acts on the atrioventricular node. 
B. It acts on the sinoatrial node. 
C. It should be avoided in atrial fibrillation. 
D. It should be avoided in patients in active bronchospasm. 
E. It can result in transient heart block. 
 
Answer: B. It acts on the sinoatrial node. 
 
Rationale: Adenosine will result in transient heart block and may be uncomfortable for patients. It can 
exacerbate bronchospasm and, in atrial fibrillation (AF) with preexcitation, it can result in 
decompensation to ventricular fibrillation. It also does not help terminate AF but is not harmful. 
 
References:  
1.) Singh S, McKintosh R. Adenosine. In: StatPearls [Internet]. Treasure Island (FL): StatPearls Publishing; 
2021 Jan. 2020 Jul 10. 
 
 
Question 14: A 63‐year‐old man presents to the emergency department (ED) 1 week after several bouts 
of diarrhea. He had tingling in his fingers and toes, had progressively difficulty walking throughout the 
day, and was uncoordinated and weak in the arms. On arrival in the ED, he was found to have bilateral 
facial weakness with weak cough and gag. He was areflexic. Pulmonary function testing revealed 
negative inspiratory force of –10 cm H2O and forced vital capacity of 1 L (10 mL/kg). The nurse noted 
pooling of secretions. Vital signs are: temperature 36.8 °C (98.2 °F), heart rate 110 beats/min, 
respiratory rate 24 breaths/min, blood pressure 160/90 mm Hg, and oxygen saturation 90% on 5 L/min 
nasal cannula. Arterial blood gas analysis shows: pH 7.37, Paco2 38 mm Hg, Pao2 75 mm Hg, HCO3 25 
mEq/L. CBC and serum chemistries are within normal limits. All of the following statements regarding 
this patient are true except 
 
A. dysautonomia is common and is associated with higher mortality. 
B. IVIg or plasma exchange is part of the therapeutic regimen. 
C. mechanical ventilation should not be delayed if pulmonary function tests show vital capacity 
less than 20 mL/kg or maximal inspiratory pressure less than –30 cm H2O and maximal 
expiratory pressure less than 40 cm H2O. 
D. steroids should be administered as immunomodulators. 
 
Answer: D. steroids should be administered as immunomodulators. 
 
Rationale: This patient has Guillain‐Barré syndrome (GBS), an autoimmune‐mediated nerve injury that 
usually presents with rapid progressive and symmetric ascending weakness. GBS is often preceded by an 
infection, in this case probably Campylobacter jejuni. GBS‐associated respiratory failure is usually 
tracked by pulmonary function testing to help make the decision to intubate. The 20‐30‐40 rule is often 

 
25
Neurocritical Care Review 
Practice Questions 
Section 2. 
 
followed: forced vital capacity less than 20 mL/kg or maximal inspiratory pressure less than –30 cm H2O 
and maximal expiratory pressure less than 40 cm H2O. If a patient meets all these criteria in addition to 
other respiratory issues (bulbar weakness, poor secretion management), intubation should not be 
delayed. IVIg or plasma exchange is the mainstay of initial treatment for GBS. Steroids are not part of 
the therapeutic regimen for GBS. 
 
References:  
1.) Greene‐Chandos D, Torbey M. Critical care of neuromuscular disorders. Continuum (Minneap Minn). 
2018 Dec;24(6):1753‐1775. 
 
2.) Wijdicks EFM, Klein CJ. Guillain‐Barré syndrome. Mayo Clin Proc. 2017 Mar;92(3):467‐479. 
 
3.) Winer JB, Hughes RA. Identification of patients at risk of arrhythmia in the Guillain‐Barré syndrome. 
Q J Med. 1988 Sep;68(257):735‐739. 
 
 
Question 15: A man presents with hypoxic respiratory failure after having an aspiration episode while 
eating dinner. He is subsequently admitted to the ICU. His wife says that he is currently undergoing 
workup for amyotrophic lateral sclerosis (ALS). All of the following are true regarding respiratory failure 
in patients with ALS except 
 
A. among patients with cranial nerve involvement, noninvasive positive pressure ventilation does 
not prolong survival.  
B. the main cause of death in patients with ALS is respiratory complications associated with 
diaphragm weakness, aspiration, or pneumonia.  
C. riluzole, a glutamatergic antagonist, has been shown to prevent respiratory failure in patients 
with ALS. 
D. tracheostomy has been found to prolong survival of patients with ALS by approximately 9 
months compared to those who refused tracheostomy. 
 
Answer: C. riluzole, a glutamatergic antagonist, has been shown to prevent respiratory failure in 
patients with ALS. 
 
Rationale: Amyotrophic lateral sclerosis (ALS) is a progressive disease that presents with progressive 
weakness in bulbar, axial, and extremity muscles with both upper and motor neuron features. The most 
common reason for death is respiratory failure associated with diaphragm weakness, aspiration, or 
pneumonia. Among patients without cranial nerve involvement, noninvasive positive pressure 
ventilation improves survival and quality of life. Those with cranial nerve involvement do not show this 
benefit. Tracheostomy was also found to have a survival benefit of 9 months for these patients. Riluzole 
was found to improve survival among patients with ALS but has no direct effect on preventing 
respiratory failure. 
 
References:  
1.) Greene‐Chandos D, Torbey M. Critical care of neuromuscular disorders. Continuum (Minneap Minn). 
2018 Dec;24(6):1753‐1775. 

 
26
Neurocritical Care Review 
Practice Questions 
Section 2. 
 
 
2.) Radunovic A, Annane D, Rafiq MK, Mustfa N. Mechanical ventilation for amyotrophic lateral 
sclerosis/motor neuron disease. Cochrane Database Syst Rev. 2013 Mar 28;(3):CD004427. 
 
3.) Bourke SC, Tomlinson M, Williams TL, Bullock RE, Shaw PJ, Gibson GJ. Effects of non‐invasive 
ventilation on survival and quality of life in patients with amyotrophic lateral sclerosis: a randomised 
controlled trial. Lancet Neurol. 2006 Feb;5(2):140‐147. 
 
4.) Sancho J, Servera E, Díaz JL, Bañuls P, Marin J. Home tracheotomy mechanical ventilation in patients 
with amyotrophic lateral sclerosis: causes, complications and 1‐year survival. Thorax. 2011 
Nov;66(11):948‐952. 
 
5.) Fang T, Al Khleifat A, Meurgey JH, et al. Stage at which riluzole treatment prolongs survival in patients 
with amyotrophic lateral sclerosis: a retrospective analysis of data from a dose‐ranging study. Lancet 
Neurol. 2018 May;17(5):416‐422. 
 
 
Question 16: In the assessment of neuromuscular weakness in the critical care setting, it is important to 
identify and delineate upper motor neuron (UMN) and lower motor neuron (LMN) signs, which will help 
in the evaluation of nontraumatic weakness. All of the following are true regarding UMN and LMN signs 
except 
 
A. hyperreflexia, increased extremity tone, and an upgoing toe are associated with UMN lesions 
(brain and spinal cord). 
B. involvement of both UMN and LMN can never occur because of distinct differentiation of UMN 
and LMN syndromes. 
C. LMN lesions localize from the anterior horn cells within the spinal cord to the muscles. 
D. sensory involvement is important in assessment of neuromuscular weakness in the ICU. 
 
Answer: B. involvement of both UMN and LMN can never occur because of distinct differentiation of 
UMN and LMN syndromes. 
 
Rationale: Neuromuscular weakness assessment in the ICU may be difficult and may be confounded by 
sedation and use of neuromuscular blockade. A thorough neurologic examination includes assessing for 
upper motor neuron (UMN) and lower motor neuron (LMN) signs, which can guide localization and 
diagnostic testing. UMNs typically localize to the brain and spinal cord while LMNs include lesions from 
the anterior horn cells within the spinal cord to the muscles. Since the anterior horn cells are within the 
spinal cord, neurologic conditions exist that may present as having both UMN and LMN involvement, 
such as amyotrophic lateral sclerosis, polio, West Nile virus, and enterovirus D68. Sensory involvement 
should be delineated because the degree and extent of involvement will also guide the localization of 
the lesion (sensory tract involvement, radiculo‐plexo‐neuropathies) even if the main concern is 
weakness. 
 
Reference:  

 
27
Neurocritical Care Review 
Practice Questions 
Section 2. 
 
1.) Caulfield AF, Flower O, Pineda JA, Uddin S. Emergency neurological life support: acute non‐traumatic 
weakness. Neurocrit Care. 2017 Sep;27(Suppl 1):29‐50. 

 
28
Neurocritical Care Review 
Practice Questions 
Section 3. 
 
Question 1: A 43‐year‐old man with no past medical history is transferred from an outside hospital for 
acute onset of drowsiness, diplopia, and slurred speech 1 hour ago. Noncontrast head CT is normal. 
Initial serum electrolytes are: sodium 138 mEq/L, potassium 3.6 mEq/L, chloride 95 mEq/L, bicarbonate 
23 mEq/L, BUN 18 mg/dL, creatinine 1.3 mg/dL. Brain CT angiogram (CTA) is ordered. Which of the 
following is the most appropriate approach to the CTA? 
 
A. Delay CTA several hours to administer N‐acetylcysteine and sodium bicarbonate to avoid 
contrast‐induced nephropathy (CIN). 
B. Perform CTA after a temporary dialysis catheter is placed to provide prophylactic renal 
replacement therapy (RRT) to prevent CIN. 
C. Perform CTA since imaging is urgently needed and there is no increased risk for RRT from CIN. 
D. Perform CTA since there is no evidence that iodinated contrast leads to creatinine elevation. 
E. Do not perform CTA because of the risk of CIN leading to RRT. 
 
Answer: C. Perform CTA since imaging is urgently needed and there is no increased risk for RRT from 
CIN. 
 
Rationale: Contemporary iso‐osmolar contrast agents have been associated with increased creatinine 
but contrast‐induced nephropathy (CIN) remains a controversial topic. There are no data to date to 
support an increased risk for renal replacement therapy from CIN, and prophylactic dialysis does not 
lead to decreased incidence of kidney injury. Urgent imaging studies requiring contrast administration 
for proper patient care should not be delayed. 
 
References:  
1.) McDonald RJ, McDonald JS, Carter RE, et al. Intravenous contrast material exposure is not an 
independent risk factor for dialysis or mortality. Radiology. 2014 Dec;273(3):714‐725. 
 
2.) Song K, Jiang S, Shi Y, Shen H, Shi X, Jing D. Renal replacement therapy for prevention of contrast‐
induced acute kidney injury: a meta‐analysis of randomized controlled trials. Am J 
Nephrol. 2010;32(5):497‐504. 
 
 
 
 
 
 
 
 
 
 
 
 
 
 

29
Neurocritical Care Review 
Practice Questions 
Section 3. 
 
Question 2: A 43‐year‐old man with a history of depression and tobacco abuse presents with loss of 
consciousness and generalized convulsions. Emergency medical services intubated him for airway 
protection in the setting of emesis and administered 10 mg IM midazolam. In the emergency 
department, 4 mg IV lorazepam is administered for ongoing convulsions followed by a 20‐mg/kg IV 
fosphenytoin load. Vital signs are notable for heart rate 115 beats/min and blood pressure 178/82 mm 
Hg. Examination is notable for left gaze deviation and rhythmic left arm and leg jerking. He has been 
seizing for approximately 40 minutes. Head CT reveals no acute intracranial process. Which of the 
following is the most appropriate next step? 
 
A. Administer another 4 mg IV lorazepam. 
B. Initiate continuous EEG monitoring. 
C. Initiate midazolam infusion with a loading dose. 
D. Load 40 mg/kg of IV levetiracetam. 
E. Obtain brain MRI with contrast. 
 
Answer: C. Initiate midazolam infusion with a loading dose. 
 
Rationale: This patient meets criteria for generalized convulsive status epilepticus (GCSE). Since his GCSE 
is refractory to appropriate emergent initial therapy (also known as initial therapy phase) with 
benzodiazepines and refractory to urgent control therapy (also known as second therapy phase) with 
fosphenytoin, he is considered to have refractory status epilepticus. The next step is to initiate 
refractory therapy (also known as third therapy phase). Because he has already received first and 
second phases of therapy, lorazepam and levetiracetam, respectively, are no longer the most 
appropriate next step. Continuous EEG monitoring would be important to assess for nonconvulsive 
status epilepticus; however, he should be treated based on clinical findings at this time. MRI might be 
indicated for workup of the seizures but should not be performed until the seizures have resolved and 
he is stabilized. 
 
References:  
1.) Brophy GM, Bell R, Claassen J, et al; Neurocritical Care Society Status Epilepticus Guideline Writing 
Committee. Guidelines for the evaluation and management of status epilepticus. Neurocrit Care. 2012 
Aug;17(1):3‐23. 
 
2.). Glauser T, Shinnar S, Gloss D, et al. Evidence‐based guideline: treatment of convulsive status 
epilepticus in children and adults: report of the Guideline Committee of the American Epilepsy Society. 
Epilepsy Curr. 2016 Jan‐Feb;16(1):48‐61. 
 
 
Question 2: A 43‐year‐old man with a history of depression and tobacco abuse presents with loss of 
consciousness and generalized convulsions. Emergency medical services intubated him for airway 
protection in the setting of emesis and administered 10 mg IM midazolam. In the emergency 
department, 4 mg IV lorazepam is administered for ongoing convulsions followed by a 20‐mg/kg IV 
fosphenytoin load. Vital signs are notable for heart rate 115 beats/min and blood pressure 178/82 mm 
Hg. Examination is notable for left gaze deviation and rhythmic left arm and leg jerking. He has been 

30
Neurocritical Care Review 
Practice Questions 
Section 3. 
 
seizing for approximately 40 minutes. Head CT reveals no acute intracranial process. Which of the 
following is the most appropriate next step? 
 
A. Administer another 4 mg IV lorazepam. 
B. Initiate continuous EEG monitoring. 
C. Initiate midazolam infusion with a loading dose. 
D. Load 40 mg/kg of IV levetiracetam. 
E. Obtain brain MRI with contrast. 
 
Answer: C. Initiate midazolam infusion with a loading dose. 
 
Rationale: This patient meets criteria for generalized convulsive status epilepticus (GCSE). Since his GCSE 
is refractory to appropriate emergent initial therapy (also known as initial therapy phase) with 
benzodiazepines and refractory to urgent control therapy (also known as second therapy phase) with 
fosphenytoin, he is considered to have refractory status epilepticus. The next step is to initiate 
refractory therapy (also known as third therapy phase). Because he has already received first and 
second phases of therapy, lorazepam and levetiracetam, respectively, are no longer the most 
appropriate next step. Continuous EEG monitoring would be important to assess for nonconvulsive 
status epilepticus; however, he should be treated based on clinical findings at this time. MRI might be 
indicated for workup of the seizures but should not be performed until the seizures have resolved and 
he is stabilized. 
 
References:  
1.) Brophy GM, Bell R, Claassen J, et al; Neurocritical Care Society Status Epilepticus Guideline Writing 
Committee. Guidelines for the evaluation and management of status epilepticus. Neurocrit Care. 2012 
Aug;17(1):3‐23. 
 
2.). Glauser T, Shinnar S, Gloss D, et al. Evidence‐based guideline: treatment of convulsive status 
epilepticus in children and adults: report of the Guideline Committee of the American Epilepsy Society. 
Epilepsy Curr. 2016 Jan‐Feb;16(1):48‐61. 
 
 
 
 
 
 
 
 
 
 
 
 
 
 

31
Neurocritical Care Review 
Practice Questions 
Section 3. 
 
Question 3: A 19‐year‐old woman is in the ICU for status epilepticus secondary to anti‐NMDA receptor 
encephalitis. She was started on propofol 3 days ago after ongoing status epilepticus despite treatment 
with IV lorazepam, IV levetiracetam, and IV lacosamide. She has been seizure free for the past 16 hours 
on continuous EEG monitoring revealing 70% burst suppression. The nurse calls the physician bedside to 
report the following vital signs: temperature 38.3 °C (100.94 °F), heart rate 41 beats/min, mean arterial 
pressure 59 mm Hg, respiratory rate 20 breaths/min, and oxygen saturation 95%. Propofol is currently 
infusing at 60 µg/kg/min. The patient is on a norepinephrine infusion of 0.8 µg/kg/min and the nurse has 
rapidly escalated the dosage over the past 2 hours. Which of the following is the most appropriate 
approach? 
 
A. Continue propofol and initiate broad‐spectrum antibiotics for sepsis. 
B. Discontinue propofol and transition to midazolam infusion. 
C. Increase propofol infusion rate. 
D. Initiate midazolam infusion. 
E. Obtain head CT and add vasopressin infusion. 
 
Answer: B. Discontinue propofol and transition to midazolam infusion. 
 
Rationale: The combination of rapidly progressive arterial hypotension and bradycardia are highly 
indicative of propofol infusion syndrome. Common manifestations include circulatory collapse, lactic 
acidosis, bradycardia, rhabdomyolysis, and hypertriglyceridemia. It is often fatal because refractory 
bradycardia can lead to asystole. Risk factors include longer duration of propofol exposure, higher 
propofol doses, younger age, and acute neurologic injury. Treatment is supportive care along with 
immediate discontinuation of propofol. If propofol is stopped, a different IV anesthetic should be used in 
its place to maintain status epilepticus cessation. Sepsis is in the differential diagnosis, and it may be 
reasonable to initiate empiric antibiotics while sepsis and propofol syndrome workups are ongoing, but 
propofol should be discontinued. If there is concern for propofol infusion syndrome, propofol should be 
stopped immediately. An additional vasopressor may be added to treat hypotension since the 
norepinephrine dose is high but the propofol should be discontinued and a different antiepileptic 
infusion started. 
 
Reference:  
1.) Nelson SE, Varelas PN. Status epilepticus, refractory status epilepticus, and super‐refractory status 
epilepticus. Continuum (Minneap Minn). 2018 Dec;24(6):1683‐1707. 
 
 
Question 4: An 84‐kg (185.2‐lb), 56‐year‐old man with chronic obstructive pulmonary disease and 
hypertension is brought to the emergency department by emergency medical services for nonsensical 
speech and right upper extremity weakness. Noncontrast head CT reveals a left parietal intracerebral 
hemorrhage measuring 1.5 × 1.9 × 2.0 cm without mass effect or midline shift. Nicardipine infusion is 
initiated for systolic blood pressure of 190 mm Hg. On admission to the neuro‐ICU, vital signs are: 
temperature 37.8 °C (100.0 °F), heart rate 84 beats/min, blood pressure 151/78 mm Hg, respiratory rate 
18 breaths/min, and oxygen saturation as measured by pulse oximetry 92% on 2 L nasal cannula. The 
nurse notifies the physician that the patient began to have forced right gaze deviation and right upper 

32
Neurocritical Care Review 
Practice Questions 
Section 3. 
 
extremity shaking progressing to a generalized tonic‐clonic seizure. When the physician arrives at the 
bedside, the patient is still seizing. Which of the following is the most appropriate course of action? 
 
A. Administer levetiracetam, 40 mg/kg IV. 
B. Administer lorazepam, 4 mg IV. 
C. Initiate emergent continuous EEG monitoring. 
D. Obtain emergent brain MRI. 
 
Answer: C. Initiate emergent continuous EEG monitoring. 
 
Rationale: The first step in this patient’s management should be administration of benzodiazepines. 
Since he has IV access, the recommended benzodiazepine is lorazepam, IV 0.1 mg/kg IV, maximum 4 mg. 
Lorazepam underdosing has been associated with a high chance of progressing to refractory status 
epilepticus, and it has not been demonstrated to be safer from an airway perspective. Based on the 
Neurocritical Care Society’s status epilepticus guidelines, an antiseizure medication should also be 
administered as part of urgent control therapy to maintain seizures cessation before EEG monitoring or 
MRI imaging. Continuous EEG monitoring is recommended if he does not return to baseline soon after 
treatment. 
 
References:  
1.) Brophy GM, Bell R, Claassen J, et al; Neurocritical Care Society Status Epilepticus Guideline Writing 
Committee. Guidelines for the evaluation and management of status epilepticus. Neurocrit Care. 2012 
Aug;17(1):3‐23. 
 
2.) Nelson SE, Varelas PN. Status epilepticus, refractory status epilepticus, and super‐refractory status 
epilepticus. Continuum (Minneap Minn). 2018 Dec;24(6):1683‐1707. 
 
 
Question 5: A 48‐year‐old woman has bacterial meningitis/ventriculitis with mild brain edema. She is in 
a coma with high‐grade papilledema and has external ventricular drains in place for continuous 
intracranial pressure monitoring, which has been controlled with cerebrospinal fluid diversion and 
sedation. Continuous EEG monitoring is in place and shows findings consistent with encephalopathy but 
no seizures. Vital signs are normal except for temperature of 39.1 °C (102.3 °F). Which of the following is 
the goal in temperature management for this patient? 
 
A. Aim for normothermia. 
B. Induce hypothermia with targeted temperature of 32 °C (89.6 °F) for prevention of secondary 
brain injury. 
C. Induce hypothermia with targeted temperature of 34 °C (93.2 °F) for prevention of secondary 
brain injury. 
D. Do not treat the fever because it is the body’s defense against infection. 
 
Answer: A. Aim for normothermia. 
 

33
Neurocritical Care Review 
Practice Questions 
Section 3. 
 
Rationale: Because of this patient’s cerebral edema and risk for secondary brain injury, fever control 
with the goal of normothermia is a reasonable approach. Elevated temperature has been shown to lead 
to worse outcomes in brain‐injured patients so the fever should be treated. Induced hypothermia is 
contraindicated in comatose patients with bacterial meningitis, after a multicenter French randomized 
control trial was stopped prematurely because of excess mortality in the hypothermia group. The 
authors concluded that moderate hypothermia did not improve outcome in patients with severe 
bacterial meningitis and may even be harmful. 
 
References: 
1.) Mourvillier B, Tubach F, van de Beek D, et al. Induced hypothermia in severe bacterial meningitis: a 
randomized clinical trial. JAMA. 2013 Nov 27;310(20):2174‐2183. 
 
2.) Meyfroidt G, Kurtz P, Sonneville R. Critical care management of infectious meningitis and 
encephalitis. Intensive Care Med. 2020 Feb;46(2):192‐201. 
 
 
Question 6: A 27‐year‐old man is admitted to the neuro‐ICU after sustaining a severe traumatic brain 
injury in a motor vehicle collision. Glasgow Coma Scale score is 3T. Head CT is significant for traumatic 
subarachnoid hemorrhage and diffuse cerebral edema consistent with diffuse axonal injury. He remains 
endotracheally intubated and on mechanical ventilation. The nurse notifies the physician that, during 
the past hour, the patient has required an escalation of norepinephrine dose and has increased urine 
output of greater than 300 mL/hr for the past 3 hours. Emergent laboratory results reveal an increase in 
serum sodium from 143 to 154 mEq/L since morning values, urine specific gravity 0.1003 and urine 
osmolality 280 mOsm/kg. In addition to close frequent monitoring of electrolytes and urine output, 
which of the following is the most appropriate next step? 
 
Add phenylephrine to maintain mean arterial pressure parameters, with isotonic crystalloid fluid boluses 
to replace urinary losses. 
Administer 1 unit packed RBCs and obtain emergent laboratory studies for acute blood loss anemia and 
disseminated intravascular coagulopathy. 
Administer desmopressin, 1 µg subcutaneously, along with isotonic crystalloid IV fluid replacement. 
Administer hypotonic crystalloid IV fluid infusion with additional boluses to replace urinary losses. 
 
Answer: C. Administer desmopressin, 1 µg subcutaneously, along with isotonic crystalloid IV fluid 
replacement. 
 
Rationale: This patient is manifesting classic signs of central diabetes insipidus (CDI), including free 
water retention (urine osmolality less than 300 mOsm/kg, urine specific gravity less than 1.005), and 
hypernatremia (sodium > 145 mEq/L) secondary to antidiuretic hormone deficiency. The mainstay of 
treating CDI in patients with critical illness with hemodynamically significant hypovolemia is 
desmopressin and isotonic fluids until volume is resuscitated. The isotonic volume replacement should 
be followed by hypotonic fluids to replace the free water deficit when hemodynamically stable. This 
patient may transiently require an additional vasopressor if maximum norepinephrine is used but 
treatment should be aimed at retaining the free water and repleting the fluids. He has hypotension due 
to CDI and not hemorrhagic shock, so blood products are not indicated. Typically, dextrose in water 

34
Neurocritical Care Review 
Practice Questions 
Section 3. 
 
fluids are used to replete deficits; however, in brain or spinal cord injury with central nervous system 
edema, enteral boluses of free water or hypotonic concentrations of IV sodium chloride infusions should 
be ued. For patients who are hemodynamically stable and unable to drink, desmopressin, 1 µg, should 
be administered subcutaneously 1‐3 times daily and free water deficit replacement with IV fluids that 
are hypotonic relative to the serum to replace urinary losses. 
 
References:  
1.) Rosenbaum BP, Steinmetz MP. Central diabetes insipidus after staged spinal surgery. Global Spine J. 
2013 Dec;3(4):257‐260. 
 
2.) Lidner G, Funk GC. Hypernatremia in critically ill patients. J Crit Care. 2013 Apr;28(2):216.e11‐
216.e20. 
 
 
Question 7: A 33‐year‐old woman is admitted to the neuro‐ICU for a Hunt and Hess grade 5, modified 
Fisher grade 4, left middle cerebral artery aneurysmal subarachnoid hemorrhage. An external 
ventricular drain is placed for hydrocephalus. On post‐bleed day 7, morning sodium level is 131 mEq/L, 
down from 142 mEq/L on admission. Which of the following 4 findings would support a diagnosis of 
cerebral salt wasting syndrome over syndrome of inappropriate antidiuretic hormone? 
 
A. Elevated BUN‐to‐creatinine ratio, elevated hematocrit, tachycardia, and metabolic alkalosis 
B. Leukocytosis, elevated hemoglobin, low urine osmolality, and tachycardia 
C. Leukopenia, azotemia, fever, and tachycardia 
D. Hypoalbuminemia, central venous pressure greater than 6 cm H2O, pitting pedal edema, and 
anemia 
 
Answer: A. Elevated BUN‐to‐creatinine ratio, elevated hematocrit, tachycardia, and metabolic alkalosis 
 
Rationale: Hyponatremia is the most common electrolyte abnormality in acute neurologic injury. 
Syndrome of inappropriate diuretic hormone (SIADH) has been reported to account for the vast majority 
of hyponatremia in patients with subarachnoid hemorrhage, but higher grade at presentation is 
associated with an increased likelihood of developing cerebral salt wasting syndrome (CSWS) over 
SIADH. Correct diagnosis is important because the treatments differ; however, distinction can be 
challenging because of their overlapping clinical presentation of hyponatremia, decreased plasma 
osmolality, increased urine sodium excretion, and urine osmolality. The main distinguishing feature is 
volume status. SIADH results in a euvolemic to hypervolemic state, while CSWS results in a hypovolemic 
state with loss of both intravascular and extracellular fluid volume. Therefore, key signs of hypovolemia 
on clinical examination, such as tachycardia, hypotension or orthostatic hypotension, weight loss, net 
negative fluid balance, and dry mucous membranes, along with laboratory findings of azotemia, 
contraction alkalosis, elevated hematocrit, hemoglobin, albumin, and BUN‐to‐creatinine ratio help make 
the diagnosis of CSWS. 
 
References:  
1.) Cui H, He G, Lv Y, et al. Inappropriate antidiuretic hormone secretion and cerebral salt‐wasting 
syndromes in neurological patients. Front Neurosci. 2019 Nov 8;13:1170. 

35
Neurocritical Care Review 
Practice Questions 
Section 3. 
 
 
2.) Verbalis JG. The curious story of cerebral salt wasting: fact or fiction? Clin J Am Soc Nephrol. 2020 
Nov 6;15(11):1666‐1668. 
 
 
Question 8: A 68‐year‐old woman with a history of coronary artery disease with prior myocardial 
infarction and hypothyroidism is admitted to the neuro‐ICU for subacute ischemic stroke in the setting 
of new‐onset atrial fibrillation with a rapid ventricular rate. Her last known well day was 3 days ago. She 
was found by her neighbors who report that during the past several months she has been confused, has 
had progressive fatigue, and always felt cold. They report that recently she has been slow to respond 
and has been staying in bed all day. Presenting vital signs are: blood pressure 90/50 mm Hg, heart rate 
54 beats/min, temperature 35.7 °C (96.3 °F), and respiratory rate 9 breaths/min. On examination, she is 
somnolent with macroglossia and edema of her hands, shins, face, and lips. Brain MRI shows a subacute 
stroke affecting the Wernicke area. Her mental status continues to decline, requiring intubation for 
airway protection. Which of the following is the most appropriate treatment for this patient? 
 
A. Bolus of 10 mL/kg crystalloid fluid followed by broad‐spectrum IV antibiotics 
B. Intramuscular epinephrine followed by IV bolus of 10 mL/kg crystalloid fluid 
C. IV hydrocortisone followed by IV liothyronine hormone loading dose 
D. IV hydrocortisone followed by IV levothyroxine hormone loading dose 
 
Answer: D. IV hydrocortisone followed by IV levothyroxine hormone loading dose 
 
Rationale: This patient’s history is concerning for decompensated hypothyroidism (previously called 
myxedema coma). Her systems of progressive fatigue, confusion, slowed mental processing, and cold 
intolerance are all consistent with untreated hypothyroidism. Also, on examination, her bradycardia, 
hypotension, hypothermia, hypoventilation, and evidence of myxedema of the tongue, hands, shins, 
lips, and face are consistent with acute decompensation, which is typically triggered by acute illness 
such as cerebrovascular events, hypoxemia, hypothermia, myocardial infarcts, hypoglycemia, infection, 
surgery, gastrointestinal hemorrhage, burns, and trauma. It can also occur with exposure to certain 
drugs such as lithium, sedatives, tranquilizers, anesthetics, narcotics, phenytoin, rifampin, diuretics, 
beta‐blockers, or amiodarone, which has direct thyrotoxic effects. Decompensated hypothyroidism 
causes multiorgan dysfunction and is life‐threating. Parenteral hydrocortisone replacement before 
thyroid hormone replacement is critical as a component of adrenal insufficiency because it can 
precipitate adrenal crisis. Either parenteral liothyronine (T3) or levothyroxine (T4) or a combination 
thereof can be used treat decompensated hypothyroidism. T4, the inactive form, is recommended for 
patients over age 65, with coronary artery disease, or at risk for arrhythmias. Because this patient had a 
prior myocardial infarction, is older than 65, and presented with atrial fibrillation with rapid ventricular 
rates, T4 is preferred over T3. Her history of symptoms of chronic untreated hypothyroidism along with 
the findings of myxedema distinguish this case as decompensated hypothyroidism as opposed to other 
diagnoses such as sepsis requiring broad‐spectrum antibiotics or anaphylaxis requiring epinephrine. 
 
Reference:  
1. ) Bridwell RE, Willis GC, Gottlieb M, Koyfman A, Long B. Decompensated hypothyroidism: a review for 
the emergency clinician. Am J Emerg Med. 2021 Jan;39:207‐212. 

36
Neurocritical Care Review 
Practice Questions 
Section 3. 
 
Question 9: A 23‐year‐old man is admitted with polytrauma including traumatic brain injury with initial 
Glasgow Coma Scale score 7 and evidence of diffuse axonal injury with hemorrhagic contusions. On day 
2, intracranial pressure (ICP) is 22 mm Hg with cerebral perfusion pressure (CPP) greater than 60 mm Hg. 
Serum electrolytes are: sodium 148 mEq/L, potassium 4.2 mEq/L, chloride 110mEq/L, bicarbonate 19 
mEq/L, BUN 42 mg/dL, and creatinine 1.3 mg/dL. On day 6, intermittent hemodialysis (iHD) is started for 
potassium elevation and volume overload. Serum electrolytes are: sodium 156 mEq/L, potassium 6.9 
mEq/L, chloride 115 mEq/L, bicarbonate 15 mEq/L, BUN 88 mg/dL, and creatinine 2.1 mg/dL. ICP is 28 
mm Hg with CPP greater than 60 mm Hg. Ninety minutes into iHD, ICP is 41 mm Hg and CPP is 55 mm 
Hg. Serum electrolytes now are: sodium 151 mEq/L, potassium 6.4 mEq/L, chloride 113 mEq/L, 
bicarbonate 18 mEq/L, BUN 55 mg/dL, and creatinine 1.9 mg/dL. Which of the following is the most 
appropriate treatment approach? 
 
A. Continue iHD and administer hypertonic saline bolus to bring sodium level to 155 mEq/L. 
B. Continue iHD and administer sodium bicarbonate bolus to bring sodium level above 155 mEq/L 
and bicarbonate level above 22 mEq/L. 
C. Discontinue iHD and start continuous renal replacement therapy (CRRT) to help maintain CPP 
greater than 60 mm Hg. 
D. Discontinue iHD and start CRRT to reduce the rate of osmotic gradient shifts due to changes in 
BUN and bicarbonate concentrations. 
E. Slow the rate of iHD to allow for optimization of CPP to greater than 60 mm Hg. 
 
Answer: D. Discontinue iHD and start CRRT to reduce the rate of osmotic gradient shifts due to changes 
in BUN and bicarbonate concentrations. 
 
Rationale: Urea is an osmotically active substance that is present in various body compartments and is 
freely diffusible but at a slower rate. Rapid removal of urea in the blood, as may occur with dialysis, can 
lead to osmotic fluid shifts between compartments and can result in elevation in intracranial pressure 
(ICP). Although hypertonic saline may be necessary to treat acute ICP elevation, intermittent 
hemodialysis (iHD) should be discontinued. The acute change is unlikely related to the rise in 
bicarbonate and would not treat the acute ICP elevation. The drop in cerebral perfusion pressure to 55 
mm Hg is less likely to be causing the acute ICP elevation and is more likely the osmotic gradient shift. 
iHD needs to be stopped, not slowed. 
 
Reference:  
1.) Lund A, Damholt MB, Wiis J, Kensen J, Strange DG, Moller K. Intracranial pressure during 
hemodialysis in patients with acute brain injury. Acta Anaesthesiol Scand. 2019 Apr;63(4):493‐499. 
 
 
Question 10: A 64‐year‐old woman is admitted to the hospital with a Hunt Hess grade 5 aneurysmal 
subarachnoid hemorrhage. She has been on a continuous 0.9% saline infusion at 75 mL/hr since arrival. 
On day 6, serum electrolytes are: sodium 141 mEq/L, potassium 3.6 mEq/L, chloride 108 mEq/L, 
bicarbonate 23 mEq/L, BUN 12 mg/dL, and creatinine 1.1 mg/dL. Central venous pressure (CVP) is 10 
mm Hg. On days 8‐9, her urine output increases and serum electrolytes are now: sodium 131 mEq/L, 
potassium 3.8 mEq/L, chloride 100 mEq/L, bicarbonate 23 mEq/L, BUN 29 mg/dL, and creatinine 1.3 
mg/dL, and CVP is 4 mm Hg. Urine osmolality is 372 mOsml/Kg and urine sodium is 98 mEq/L. On 

37
Neurocritical Care Review 
Practice Questions 
Section 3. 
 
physical examination, there are no signs of volume overload. Which of the following is the most 
appropriate approach to her management? 
 
A. Switch IV fluids to a higher‐concentration sodium solution (hypertonic saline) to increase serum 
sodium concentration and avoid hypovolemia. 
B. Stop IV fluids and start fludrocortisone to increase serum sodium concentration. 
C. Stop IV fluids and implement fluid restriction to increase serum sodium concentration. 
D. Stop IV fluids and administer salt tablets to increase serum sodium concentration. 
E. Obtain central nervous system imaging because she has developed central diabetes insipidus. 
 
Answer: A. Switch IV fluids to a higher‐concentration sodium solution (hypertonic saline) to increase 
serum sodium concentration and avoid hypovolemia. 
 
Rationale: Cerebral salt wasting (CSW) can be clinically similar to syndrome of inappropriate antidiuretic 
hormone (SIADH) and is mainly distinguished from it by evidence of volume contraction. In patients with 
subarachnoid hemorrhage. It is widely accepted that volume depletion should be avoided because of 
concern for vasospasm and delayed cerebral ischemia, so fluids should not be stopped. Central diabetes 
insipidus would present with volume depletion and elevated sodium due to the loss of free water. 
 
Reference:  
1.) Rabinstein AA, Bruder N. Management of hyponatremia and volume contraction. Neurocrit 
Care. 2011 Sep;15(2):354‐360. 
 
 
Question 11: A 70‐year‐old woman with chronic kidney disease stage 3 is admitted for refractory status 
epilepticus. On day 4, intermittent hemodialysis (iHD) is started for hyperkalemia and acidosis. She is on 
propofol, levetiracetam, and lacosamide. Which of the following is the most important consideration 
regarding the initiation of iHD? 
 
A. Levetiracetam and lacosamide should be re‐dosed with decreased doses of each medication and 
no further post‐iHD doses. 
B. Levetiracetam and lacosamide should be re‐dosed with supplemental doses of each 
administered post‐iHD. 
C. Lacosamide only should be re‐dosed with supplemental dose administered post‐iHD. 
D. Levetiracetam only should be re‐dosed with a supplemental dose administered post‐iHD. 
E. No changes to either medication are necessary at this point since they are highly protein bound 
and of large molecular weight. 
 
Answer: B. Levetiracetam and lacosamide should be re‐dosed with supplemental doses of each 
administered post‐iHD. 
 
Rationale: Lacosamide and levetiracetam are not highly protein bound and are subject to removal by 
dialysis. Careful attention to drug levels should be paid for any patient undergoing dialysis where 
fluctuations in medications can lead to clinically significant events. 
 

38
Neurocritical Care Review 
Practice Questions 
Section 3. 
 
Reference:  
1.) Rodríguez KAM, Benbadis SR. Managing antiepileptic medication in dialysis patients. Curr Treat 
Options Neurol. 2018 Sep 27;20(11):45. 
 
 
Question 12: A 67‐year‐old woman with a history of atrial fibrillation and on warfarin is admitted to the 
neuro‐ICU with an acute spontaneous intracerebral hemorrhage. INR is 4.3. Which of the following 
provides the best long‐term reversal of warfarin? 
 
A. 4‐factor prothrombin complex concentrate 
B. Factor VIIa 
C. Fresh frozen plasma 
D. Vitamin K, 10 mg IV 
 
Answer: D. Vitamin K, 10 mg IV 
 
Rationale: This patient has supratherapeutic warfarin and has a life‐threatening bleed that necessitates 
reversal. Warfarin inhibits vitamin K‐dependent factors II, VII, IX, and X. Therefore, reversal is aimed at 
repleting these factors with either fresh frozen plasma or prothrombin complex concentrate; however, 
the repletion of the factors is temporary, so vitamin K, 10 mg IV, must be administered at the same time 
for long‐term reversal. Factor VIIa offers an incomplete reversal because it repletes only factor VII. 
 
Reference:  
1.) Tomaselli GF, Mahaffey KW, Cuker A, et al. 2020 ACC expert consensus decision pathway on 
management of bleeding in patients on oral anticoagulants: a report of the American College of 
Cardiology Solution Set Oversight Committee. J Am Coll Cardiol. 2020 Aug 4;76(5):594‐622. 
 
 
Question 13: Which of the following is an independent risk factor for traumatic brain injury‐related 
coagulopathy? 
 
A. Diffuse axonal injury 
B. Extensor posturing on examination 
C. Hypertension on admission 
D. Injury severity scale score greater than or equal to 16 
 
Answer: D. Injury severity scale score greater than or equal to 16 
 
Rationale: Coagulopathy in traumatic brain injury (TBI) is caused by the release of tissue thromboplastin 
and activation of fibrinolysis. Incidence occurs in more than 30% of patients with TBI, especially 
penetrating injuries, and is a cause of preventable death. Independent risk factors for TBI‐related 
coagulopathy include Glasgow Coma Scale score of less than or equal to 8, injury severity scale score 
greater than or equal to 16, hypotension on admission, cerebral edema, subarachnoid hemorrhage, and 
midline shift. Laboratory findings show prolonged prothrombin time and elevated INR, with or without 
thrombocytopenia. 

39
Neurocritical Care Review 
Practice Questions 
Section 3. 
 
Reference: 
1.) Zhang J, Jiang R, Liu L, Watkins T, Zhang F, Dong JF. Traumatic brain injury‐associated coagulopathy. J 
Neurotrauma. 2012 Nov 20;29(17):2597‐2605. 
 

40
Neurocritical Care Review 
Practice Questions 
Section 4. 
 
Question 1: During induction of a patient with preeclampsia for a vaginal delivery, the patient has an 
eclamptic seizure with tonic‐clonic movements. She has finished her initial 6‐g IV bolus of magnesium 
sulfate and has been receiving a continuous infusion at 2 g/hr. Which of the following is the most 
appropriate management approach? 
 
A. Apply supplemental oxygen, administer another IV bolus of magnesium sulfate, and continue 
the infusion. 
B. Perform emergent cesarean delivery immediately after the seizure. 
C. Consult neurology for continuous EEG monitoring during labor. 
D. Intubate for airway protection and start a midazolam infusion. 
 
Answer: A. Apply supplemental oxygen, administer another IV bolus of magnesium sulfate, and continue 
the infusion. 
 
Rationale: Eclamptic seizures are tonic‐clonic seizures that usually resolve quickly and usually are not 
repeated. IV or intramuscular magnesium sulphate is used for both prophylaxis in patients with 
preeclampsia and treatment in patients with eclampsia. There is rarely an indication to treat with 
standard antiepileptic medications since eclamptic seizures resolve quickly. Although relatively 
uncommon, if this patient has a second or third seizure, a repeat bolus of 4‐ to 6‐g IV magnesium 
sulphate should be administered. If seizure activity continues despite this aggressive magnesium 
therapy, the clinician should look for another potential source of seizures with radiographic modalities 
including brain CT and continuous EEG monitoring but these may not be required if seizures subside 
with an additional dose of magnesium and delivery. A midazolam infusion would be required only for 
refractory seizures despite magnesium. Cesarean delivery is indicated only for obstetric reasons and is 
not required for seizures. Intubation is rarely required in a patient with eclampsia unless she has 
recurrent and refractory seizures and her airway is then compromised. 
 
Reference:  
1.) Patil MM. Role of neuroimaging in patients with atypical eclampsia. J Obstet Gynaecol India. 2012 
Oct;62(5):526‐530. 
 
 
Question 2: A 40‐year‐old woman at 20 weeks’ gestation is brought to the emergency department by 
emergency medical services after having right‐sided weakness and dysarthria for the past 2 hours. She 
has a past medical history of hypertension and type 1 diabetes mellitus. Initial brain CT does not show 
any acute changes. Which of the following is a consideration about the treatment approach? 
 
A. Emergent delivery should be considered. 
B. It is unlikely that she has an acute stroke since the CT is negative, but close observation is 
required. 
C. Tissue plasminogen activator cannot be administered because she is pregnant. 
D. She may be a candidate for invasive intra‐arterial mechanical thrombectomy. 
 
Answer: D. She may be a candidate for invasive intra‐arterial mechanical thrombectomy. 
 

41
Neurocritical Care Review 
Practice Questions 
Section 4. 
 
Rationale: This patient’s risk factors and clinical findings are concerning for acute ischemic stroke. 
Thrombectomy has been used during pregnancy without complications. Although several reports exist 
of pregnant patients successfully treated with IV tissue plasminogen activator (tPA) for acute ischemic 
stroke, pregnancy remains a relative exclusion criterion for IV tPA despite acceptable maternal and fetal 
hemorrhagic risks and should be considered. Ischemic changes are not usually seen on brain CT after 
only 2 hours. This lack of CT findings precludes neither tPA treatment nor thrombectomy. Three stages 
are used to describe the CT changes seen in acute ischemic stroke: acute (< 24 hours), subacute (24 
hours to 5 days) and chronic (weeks). Acute stroke represents cytotoxic edema, and early ischemic CT 
changes can be subtle. These early changes result from intracellular edema with loss of the gray‐white 
matter interface and effacement of the cortical sulci. A thrombus in the proximal middle cerebral artery 
is sometimes seen in the acute phase and appears as hyperattenuation. A subacute stroke represents 
vasogenic edema, with greater mass effect, hypoattenuation, and well‐defined margins. Chronic strokes 
are associated with loss of brain tissue and are hypoattenuating. Delivery would not be considered at 20 
weeks’ gestation since the fetus is not viable. 
 
References:  
1.) Tversky S, Libman RB, Reppucci ML, Tufano AM, Katz JM. Thrombolysis for ischemic stroke during 
pregnancy: a case report and review of the literature. J Stroke Cerebrovasc Dis. 2016 Oct;25(10):e167‐
e170. 
 
2.) Aaron S, Shyamkumar NK, Alexander S, et al. Mechanical thrombectomy for acute ischemic stroke in 
pregnancy using the penumbra system. Ann Indian Acad Neurol. 2016 Apr‐Jun;19(2):261‐263. 
 
3.) Birenbaum D, Bancroft LW, Felsberg GJ. Imaging in acute stroke. West J Emerg Med. 2011 
Feb;12(1):67‐76. 
 
 
Question 3: A 27‐year‐old woman who is gravida 3, para 2 at 38 weeks’ gestation presents with a large 
intracerebral hemorrhage (ICH). She has a history of drug abuse and has continued to use cocaine and 
heroin throughout her pregnancy. Which of the following best describes hemorrhage in pregnancy? 
 
A. Low‐dose aspirin prophylaxis reduces the risk of ICH in patients with preeclampsia. 
B. The most common etiology of hemorrhage in pregnancy is disseminated intravascular 
coagulation. 
C. Pregnancy increases the risk of ischemic strokes but not hemorrhagic strokes. 
D. The risk of death is higher during pregnancy from hemorrhagic stroke than from ischemic stroke. 
 
Answer: D. The risk of death is higher during pregnancy from hemorrhagic stroke than from ischemic 
stroke. 
 
Rationale: Intracranial hemorrhages (ICH) are classified anatomically as extradural, subdural, 
subarachnoid, or intracerebral. Noncontrast brain CT should be the initial radiographic examination if 
hemorrhage is suspected. It is advisable to limit the amount of ionizing radiation received during 
pregnancy. It is estimated that a noncontrast head CT results in shielded fetal radiation exposure of less 
than 0.005 mGy, which is significantly less than the 100 mGy that is considered the threshold for the 

42
Neurocritical Care Review 
Practice Questions 
Section 4. 
 
development of fetal malformations from radiation exposure. The incidence of adverse fetal effects with 
contrast media appears low, although there is a small risk of neonatal hypothyroidism since iodine‐
based contrast media may cross the placenta. Aneurysmal subarachnoid hemorrhage is the main cause 
of ICH during pregnancy and in the postpartum period. The highest incidence of ICH occurs during the 
third trimester and up to 12 weeks postpartum. Rupture of an arteriovenous malformation, 
hypertension, and venous occlusion are sources of intracranial bleeding in pregnancy. Trauma can result 
in extradural and subdural hemorrhages and some intracerebral hemorrhages. Although the increased 
risk for stroke during pregnancy is often attributed to a hypercoagulable state leading to ischemic 
strokes, most deaths due to maternal stroke are from ICH. It has been shown that many of these deaths 
could have been prevented by more aggressive treatment, including aggressive control of blood 
pressure and quicker transfer to higher levels of care. Blood pressure control and correction of 
coagulopathy reduce the risk of ICH in patients with preeclampsia. Low‐dose aspirin has no benefit. 
 
References:  
1.) Aoyama K, Ray JG. Pregnancy and risk of intracerebral hemorrhage. JAMA Netw Open. 2020 Apr 
1;3(4):e202844. 
 
2.) Bartsch E, Medcalf KE, Park AL, Ray JG; High Risk of Pre‐eclampsia Identification Group. Clinical risk 
factors for pre‐eclampsia determined in early pregnancy: systematic review and meta‐analysis of large 
cohort studies. BMJ. 2016 Apr 19;353:i1753. 
 
3.) Katsuragi S, Tanaka H, Hasegawa J, et al; Maternal Death Exploratory Committee in Japan and Japan 
Association of Obstetricians and Gynecologists. Analysis of preventability of stroke‐related maternal 
death from the nationwide registration system of maternal deaths in Japan. J Matern Fetal Neonatal 
Med. 2018 Aug;31(16):2097‐2104. 
 
4.) Fairhall JM, Stoodley MA. Intracranial haemorrhage in pregnancy. Obstet Med. 2009 Dec;2(4):142‐
148 
 
 
Question 4: A 78‐year‐old man had a cardiac arrest at home. Emergency medical services performed 
cardiac compressions for 30 minutes at the scene before return of spontaneous circulation. Intubation 
was difficult, but a successful airway was finally obtained after multiple attempts. In the emergency 
department (ED), he has no motor activity to deep painful stimuli and both pupils are fixed and dilated. 
He has minimal respiratory efforts. When should clinical brain death examination be performed? 
 
A. After brain CT is obtained 
B. After cessation of respirations 
C. Immediately after admission to ICU 
D. Immediately in the ED 
 
Answer: B. After cessation of respirations 
 
Rationale: Although this patient has no motor responses, he still has minimal respiratory effort. Apnea is 
an absolute criteria for brain death. A complete neurologic examination should be performed and, if 

43
Neurocritical Care Review 
Practice Questions 
Section 4. 
 
brain death is suspected based on the neurologic examination, the clinician should perform the brain 
death examination. Once the patient becomes apneic and all other criteria are met, the brain death 
examination can be performed. The brain death examination is based on clinical criteria and not on 
radiographic findings; however, radiographic evidence of a devastating brain injury with herniation or 
cerebral edema with loss of gray‐white matter differentiation would prompt the clinician to perform a 
complete neurologic examination. 
 
References: 
1.) Machado C. Diagnosis of brain death. Neurol Int. 2010 Jun 21;2(1)e2. 
  
2.) Chakraborty S, Symons SP, Chapman M, Aviv RI, Fox AJ. Diffuse ischemia in noncontrast computed 
tomography predicts outcome in patients in intensive care unit. Can Assoc Radiol J. 2012 May;63(2):129‐
134. 
 
 
Question 5: A 65‐year‐old woman is admitted to the ICU after an extensive subarachnoid hemorrhage. 
She is a not a surgical candidate for repair of the ruptured cerebral aneurysm because she meets the 
criteria for brain death by clinical examination. Commonly expected physiologic changes after brain 
death can include 
 
A. cardiac arrhythmias 
B. cerebral salt wasting  
C. increased antidiuretic hormone 
D. increased cortisol 
E. increased thyroid hormone 
 
Answer: A. cardiac arrhythmias 
 
Rationale: Hemodynamic changes can be seen after brain death and can manifest as hypotension, 
myocardial dysfunction with cardiac ischemia and arrhythmias, pulmonary edema, and coagulation 
abnormalities. Cerebral salt wasting can be seen in subarachnoid hemorrhage in which there is loss of 
high volumes of sodium and water leading to hypovolemic hyponatremia in brain death, and diabetes 
insipidus is common due to decreased antidiuretic hormone resulting in loss of free water and 
hypernatremia. In addition to diabetes insipidus, other hypothalamic‐pituitary abnormalities result in 
several endocrine disorders causing important metabolic and hemodynamic alterations including 
decreased thyroid function, catecholamine depletion with decreased cortisol and adrenocorticotropic 
hormone, and hypothermia. 
 
References:  
1.) Domi R, Sula H, Ohri I, Laho H. Pathophysiologic changes after brain death and organ preservation: 
the intensivist’s and anesthesiologist’s role. J Anesthes Clin Res. 2013 Jan;4(3):302. doi:10.4172/2155‐
6148.1000302 
 
 

44
Neurocritical Care Review 
Practice Questions 
Section 4. 
 
Question 6: A 67‐year‐old woman is admitted to the ICU after an anterior wall myocardial infarction. 
Before hospital admission, she had a prolonged cardiac arrest with return of spontaneous circulation 
after 30 minutes. Which of the following would preclude the clinician from performing a bedside clinical 
brain death examination? 
 
A. Extensor posturing 
B. Hyperthermia 
C. Presence of spinal reflexes  
D. Variable blood pressure readings 
 
Answer: A. Extensor posturing 
 
Rationale: Brain death testing requires the absence of cortical and brainstem function. This includes loss 
of motor response to noxious stimuli, loss of cranial nerve function, and apnea. Hypothermia (< 36 °C 
[96.8 °F]) is a contraindication to brain death testing since severe hypothermia can often mimic the 
profound clinical changes seen in brain death. Efforts to warm the patient are appropriate to allow for 
brain death testing. The presence of spinal reflexes is not an exclusion for brain death testing. Spinal 
reflexes, including deep tendon reflexes, plantar flexion, plantar responses, muscle stretch reflexes, 
abdominal reflexes, and finger jerks may be present with brain death. Spinal reflexes have been 
reported in up to 75% of patients declared brain dead. Patients who are critically ill may exhibit variable 
increases and decreases in blood pressure, but this variability does not preclude brain death testing if 
minimum criteria for testing are met. Untreated hypotension (systolic blood pressure ≤ 100 mm Hg) 
precludes brain death testing although treatment with fluids and/or vasopressor support to increase 
blood pressure is permissible; however, significant hemodynamic instability may prevent the clinician 
from performing the apnea test because of the risk of worsening hemodynamics. 
 
References:  
1.) Beckmann YT, Ciftci Y, Incesu TK, Secil Y, Akhan G. Spinal reflexes in brain death. Acta Neurol Belg. 
2014 Dec;114(4):303‐306. 
 
2.) Han SG, Kim GM, Lee KH, Chung CS, Jung KY. Reflex movements in patients with brain death: a 
prospective study in a tertiary medical center. J Korean Med Sci. 2006 Jun;21(3):588‐590. 
 
 
Question 7: An 86‐year‐old woman is brought to the emergency department by her daughter for new‐
onset lethargy. She opens her eyes but is nonverbal, does not follow commands, has reactive pupils, and 
withdraws all extremities to pain. She had a ventriculoperitoneal shunt placed 20 years ago for 
aqueductal stenosis. CT shows hydrocephalus. Which of the following is the most appropriate next step? 
 
A. EEG 
B. MRI 
C. Shunt study 
D. External ventricular drain placement 
E. Antibiotics 
 

45
Neurocritical Care Review 
Practice Questions 
Section 4. 
 
Answer: D. External ventricular drain placement 
 
Rationale: The presence of hydrocephalus in a patient with a ventriculoperitoneal shunt indicates a 
shunt malfunction or infection or both. Placement of an external ventricular drain (EVD) is both 
therapeutic for cerebrospinal fluid (CSF) diversion and diagnostic since it allows for collection of CSF to 
be tested for infection. Based on the CSF results, a determination can be made about whether 
antibiotics are indicated. If there are signs of sepsis or the CSF profile is abnormal after EVD placement, 
it would be appropriate to initiate antibiotics. Because of this patient’s decline on examination and new 
hydrocephalus on CT, a shunt study is not needed before CSF diversion. If she does not improve after 
EVD placement and there is no concern for meningitis, it is appropriate to consider an EEG or MRI to 
evaluate for other sources of encephalopathy. 
 
Reference:  
1.) Rodis I, Mahr CV, Frhrenbach MK, et al. Hydrocephalus in aqueductal stenosis: a retrospective 
outcome analysis and proposal of subtype classification. Childs Nerv Syst. 2016 Apr;32(4):617‐627. 
 
 
Question 8: A 60‐year‐old woman with headache, nausea, and vomiting is diagnosed with 
noncommunicating hydrocephalus. An external ventricular drain (EVD) was placed and she was found to 
have high intracranial pressures (ICPs) on the monitor, which is set to measure ICP in mm Hg. Which of 
the following is the conversion factor used to convert cm H2O for the EVD setting into mm Hg? 
 
A. 0.7 cm H2O = 1 mm Hg 
B. 1 cm H2O = 0.7 mm Hg 
C. 1 cm H2O = 1.36 mm Hg 
D. 1 mm H2O = 0.7 mm Hg 
 
Answer: B. 1 cm H2O = 0.7 mm Hg 
 
Rationale: The density of mercury is 13.6 times that of water and cerebrospinal fluid density is close to 
that of water. The conversion is 1 mm Hg = 1.36 cm H2O or 1 cm H2O = 0.735 mm Hg. 
 
Reference:  
1.) Greenberg MS. Handbook of Neurosurgery. 7th ed. Thieme; 2010;870‐871. 
 
 
Question 9: Which of the following is a contraindication to cerebrospinal fluid (CSF) diversion? 
 
A. Idiopathic intracranial hypertension 
B. Intracranial hypotension 
C. Increased intracranial pressure 
D. Normal‐pressure hydrocephalus 
E. Persistent CSF leak from a skull base defect 
 
Answer: B. Intracranial hypotension 

46
Neurocritical Care Review 
Practice Questions 
Section 4. 
 
Rationale: Intracranial hypotension caused by cerebrospinal fluid (CSF) hypovolemia can lead to altered 
mentation and transtentorial herniation if severe, and mild forms can lead to symptoms such as 
positional headaches. Diverting more CSF can transform a mild CSF hypovolemia to severe intracranial 
hypotension with symptoms and signs that can sometimes be difficult to distinguish from intracranial 
hypertension except that it is characterized by low intracranial pressure (ICP). Increased ICP is an 
indication for CSF diversion. Severe idiopathic intracranial hypertension can be treated with shunt 
placement for CSF diversion. Normal‐pressure hydrocephalus can be evaluated for improvement with 
placement of temporary CSF diversion in the form of a lumbar drain and then permanent shunt 
placement if the patient showed signs of improvement with temporary diversion. Persistent CSF leak 
after repairing the skull base defect can be treated with placement of a temporary or permanent CSF 
diversion in the form of a shunt. 
 
References:  
1.) Komotar RJ, Mocco J, Ransom ER, et al. Herniation secondary to critical postcraniotomy 
cerebrospinal fluid hypovolemia. Neurosurgery. 2005 Aug;57(2):286‐292; discussion 286‐292.  
 
2.) Winn R. Youmans and Winn Neurological Surgery. 7th ed. Vol 2. Elsevier; 2016:1602‐1613. 
 
 
Question 10: Which of the following can be used safely for treating diffuse spasticity? 
 
A. Botulinum toxin injection 
B. Dorsal root entry zone lesioning 
C. High‐dose oral baclofen 
D. Intrathecal baclofen 
E. Neurotomies 
 
Answer: D. Neurotomies 
 
Rationale: Intrathecal baclofen acts at the receptors in the dorsal gray matter of the spinal cord and is 
used for diffuse spasticity. Intrathecal administration helps concentrate the drug locally. If oral systemic 
baclofen is used, the amount needed to achieve the same concentration in the spinal cord would lead to 
altered mentation due to the depressant features on the brain. Neurotomies and botulinum toxin 
injections are used for focal spasticity. When spasticity affects an entire limb such as in paraplegia and 
hemiplegia, dorsal root entry zone lesioning may be an option. 
 
Reference: 
1.) Winn R. Youmans and Winn Neurological Surgery. 7th ed. Vol 1. Elsevier; 2016:729‐738. 
 
 
 
 
 

47
Neurocritical Care Review 
Practice Questions 
Section 4. 
 
Question 11: A 38‐year‐old man who is being treated with chimeric antigen receptor T‐cell therapy for 
diffuse large B‐cell lymphoma is admitted to the ICU with encephalopathy and seizures. Which of the 
following is the most likely diagnosis? 
 
A. Brain metastases 
B. Fungal meningitis 
C. Immune effector cell‐associated neurotoxicity syndrome 
D. Intracranial hemorrhage 
E. Limbic encephalitis 
 
Answer: C. Immune effector cell‐associated neurotoxicity syndrome 
 
Rationale: Chimeric antigen receptor T‐cell therapy can precipitate immune effector cell‐associated 
neurotoxicity syndrome, a toxic encephalopathy that results from cytokine and chemokine release. 
Patients with systemic lymphoma can have central nervous system metastases, but this is not the most 
likely diagnosis. Similarly, limbic encephalitis should be in the differential diagnosis but is not the most 
likely diagnosis. This patient’s history does not suggest an infectious process such as fungal meningitis or 
focal neurologic deficits suggestive of stroke or intracranial hemorrhage. 
 
Reference:  
1.) Neelapu SS. Managing the toxicities of CAR T‐cell therapy. Hemat Oncol. 2019 Jun;37 Suppl 1:48‐52. 
 
 
Question 12: A patient with a left frontal glioblastoma is admitted to the ICU with new‐onset seizures 
and disease progression. Vital signs are: temperature 37.2 °C (99.0 °F), blood pressure 134/82 mm Hg, 
heart rate 86 beats/min, and oxygen saturation 97% on room air. On examination, the patient is awake, 
alert, and responding appropriately to questions with symmetric motor strength throughout. In addition 
to starting an antiepileptic, which of the following medications should be administered next? 
 
A. Broad‐spectrum antibiotics 
B. Hypertonic saline 
C. Mannitol 
D. Steroids 
E. Trimethoprim‐sulfamethoxazole 
 
Answer: D. Steroids 
 
Rationale: Steroids improve vasogenic brain edema surrounding intracranial tumors and are beneficial 
in patients with edema‐associated neurologic deficits such as this patient with new‐onset seizures. 
Dexamethasone is typically used because of its long half‐life and low mineralocorticoid activity. The 
patient’s history does not suggest a current infectious process, so prophylactic antibiotics are not 
indicated. Examination findings and vital signs are not concerning for elevated intracranial pressure at 
this time, so neither hypertonic saline nor mannitol are indicated. 
 
 

48
Neurocritical Care Review 
Practice Questions 
Section 4. 
 
Reference:  
1.) Colman H. Adult gliomas. Continuum (Minneap Minn). 2020 Dec;26(6):1452‐1475. 
 
 
Question 13: A 58‐year‐old man with lung cancer and atrial fibrillation (treated at home with aspirin) 
presents with back pain and lower extremity weakness that has worsened during the past 3 days. On 
examination, his bilateral lower extremities have symmetric decreased sensation. He can lift each leg off 
the bed but is unable to hold them there. Deep tendon reflexes are hyperreflexic. Which of the following 
is the most likely diagnosis? 
 
A. Extradural metastasis 
B. Guillain‐Barré syndrome 
C. Intramedullary metastasis 
D. Spinal cord hemorrhage 
E. Spinal cord infarction 
 
Answer: A. Extradural metastasis 
 
Rationale: Given this patient’s history of lung cancer and subacute onset of back pain and paraparesis, 
the most likely diagnosis is an extradural metastasis causing spinal cord compression. While Guillain‐
Barré syndrome can present with lower extremity weakness and sensory findings, the presentation is 
commonly progressive ascending weakness and an absence of deep tendon reflexes on examination. 
Intramedullary tumors are typically ependymomas and astrocytomas; intramedullary metastases are 
uncommon in lung cancer. A spinal cord hemorrhage or spinal cord infarct would present with sudden 
onset of symptoms. 
 
References:  
1.) Kumar N, Tan WLB, Wei W, Vellayappan BA. An overview of the tumors affecting the spine: inside to 
out. Neuroonc Pract. 2020 Nov 18;7(Suppl 1): i10‐i17. 
 
 
Question 14: A 45‐year‐old woman with an ischemic stroke undergoes thrombectomy. During the 
procedure, the surgeon places a stent and uses cangrelor. The patient returns to the ICU on a cangrelor 
infusion and requires an oral P2Y12 agent. The most appropriate recommendation is to initiate 
 
A. clopidogrel 4 hours after the cangrelor infusion is stopped 
B. clopidogrel at the same time the cangrelor infusion is stopped 
C. prasugrel 2 hours before the cangrelor infusion is stopped 
D. ticagrelor 30 minutes after the cangrelor infusion is stopped 
 
Answer: B. clopidogrel at the same time the cangrelor infusion is stopped 
 
Rationale: The most appropriate recommendation is to initiate clopidogrel at the same time the 
cangrelor infusion is stopped because it avoids the clopidogrel‐cangrelor drug interaction and minimizes 
the time the patient is without platelet inhibition. Clopidogrel has a drug interaction with cangrelor that 

49
Neurocritical Care Review 
Practice Questions 
Section 4. 
 
may make the clopidogrel less effective if administered during a cangrelor infusion. It is inappropriate to 
wait 4 hours because cangrelor has an extremely short half‐life and will leave the patient without 
antiplatelet effects. Prasugrel has a drug interaction with cangrelor that may make prasugrel less 
effective if administered during a cangrelor infusion so it should be administered at the same time the 
cangrelor infusion is stopped. Ticagrelor is the only P2Y12 that does not have a drug interaction with 
cangrelor so it may be administered either during the cangrelor infusion or immediately when stopping 
it. It is inappropriate to wait 30 minutes because cangrelor has an extremely short half‐life and will leave 
the patient without antiplatelet effects. 
 
Reference:  
NA 
 
 
Question 15: Which of the following commonly used antiepileptic drug acts as an inducer of other 
drugs? 
 
A. Valproic acid  
B. Levetiracetam  
C. Lacosamide  
D. Phenytoin 
 
Answer: D. Phenytoin 
 
Rationale: Phenytoin is known to induce CYP 3A4, 1A2, and UGT, leading to subtherapeutic 
concentrations of drugs dependent on this metabolism so it may require dose increases. Valproic acid is 
an inhibitor and could lead to supratherapeutic concentrations of the drugs it inhibits and may require 
dose decreases. Levetiracetam does not have any appreciable drug interactions. Lacosamide does not 
have any appreciable drug interactions. 
 
References:  
NA 
 
 
Question 16: A 76‐year‐old man presents for status epilepticus management. He is initially appropriately 
managed with midazolam and valproic acid (VPA), resulting in termination of status. After several days 
of titration and adjustments, his seizure medications are VPA, lacosamide, and levetiracetam. Several 
days later, he becomes progressively altered, requiring intubation for airway protection. Which of the 
following is the most likely cause? 
 
A. VPA‐induced hyperammonemia 
B. Drug interaction with meropenem leading to toxic supratherapeutic VPA concentrations 
C. Neurotoxicity from levetiracetam administration via IV bolus instead of 30‐minute infusion 
D. Drug interaction between lacosamide and midazolam leading to accumulation of midazolam in 
adipose tissue 
 

50
Neurocritical Care Review 
Practice Questions 
Section 4. 
 
Answer: A. VPA‐induced hyperammonemia 
 
Rationale: Valproic acid (VPA)‐induced hyperammonemia is a dose‐independent side effect. VPA impairs 
clearance of ammonia via carnitine deficiencies. Drug interactions between VPA and carbapenems lead 
to subtherapeutic levels of VPA. This drug interaction cannot be overcome with dose increases of VPA. 
Levetiracetam is safe to administer via IV bolus. There is no drug interaction between lacosamide and 
midazolam. 
 
Reference: 
NA 
 
 
Question 17: A 53‐year‐old man presents with vision decline and is found to 
have bitemporal visual field deficit. MRI is shown on the next slide. Which of the 
following combinations of presenting signs and symptoms would be concerning 
for pituitary apoplexy? 
 
A. Blindness, tumor growth, and cerebrospinal fluid (CSF) leak 
B. CSF leak, ophthalmoplegia, and sinusitis 
C. Epistaxis, headache, and lethargy 
D. Ophthalmoplegia, tumor infarction, and tumor hemorrhage 
E. Tumor infarction, epistaxis, and death 
 
Answer: D. Ophthalmoplegia, tumor infarction, and tumor hemorrhage 
 
Rationale: The MRI demonstrates a large pituitary mass. Pituitary apoplexy is an emergent condition in 
which there is an acute infarct and hemorrhage in the pituitary gland. Presenting symptoms can include 
sudden headache, ophthalmoplegia, rapid vision decline, hypopituitarism, lethargy, and death. Most 
occur in a preexisting pituitary tumor and occur spontaneously, although there have been reports of 
precipitating events such as surgery, use of anticoagulants, traumatic brain injury, and pregnancy or 
treatment with estrogens. Surgical decompression and endocrine hormone replacement are the 
mainstays of treatment. Neither cerebrospinal fluid leak nor epistaxis occurs in apoplexy. 
 
References: 
1.) Semple PL, Jane JA Jr, Laws ER Jr. Clinical relevance of precipitating factors in pituitary apoplexy. 
Neurosurgery. 2007 Nov;61(5):956‐961; discussion 961‐962. 
 
2.) Semple PL, Webb MK, de Villiers JC, Laws ER Jr. Pituitary apoplexy. Neurosurgery. 2005;56(1):65‐72; 
discussion 72‐73. 
 
3.) Singh TD, Valizadeh N, Meyer FB, Atkinson JL, Erickson D, Rabinstein AA. Management and outcomes 
of pituitary apoplexy. J Neurosurg. 2015 Jun;122(6):1450‐1457. 
 
 
Question 18: Which of the following vasopressors has/have beta receptor activity? 

51
Neurocritical Care Review 
Practice Questions 
Section 4. 
 
A. Norepinephrine only 
B. Norepinephrine and epinephrine only 
C. Norepinephrine, epinephrine, and dopamine only 
D. Norepinephrine, epinephrine, dopamine, and phenylephrine 
 
Answer: D. Norepinephrine, epinephrine, dopamine, and phenylephrine 
 
Rationale: Norepinephrine, epinephrine, and dopamine (dose dependent) all have varying degrees of 
beta receptor activity. Phenylephrine is the only catecholamine that does not have any beta receptor 
activity. 
 
Reference:  
NA 

52

You might also like